收藏 分享(赏)

上海市2013届高三一模考试英语汇编 阅读题 WORD版含答案.doc

上传人:高**** 文档编号:9036 上传时间:2024-05-23 格式:DOC 页数:58 大小:1.01MB
下载 相关 举报
上海市2013届高三一模考试英语汇编 阅读题 WORD版含答案.doc_第1页
第1页 / 共58页
上海市2013届高三一模考试英语汇编 阅读题 WORD版含答案.doc_第2页
第2页 / 共58页
上海市2013届高三一模考试英语汇编 阅读题 WORD版含答案.doc_第3页
第3页 / 共58页
上海市2013届高三一模考试英语汇编 阅读题 WORD版含答案.doc_第4页
第4页 / 共58页
上海市2013届高三一模考试英语汇编 阅读题 WORD版含答案.doc_第5页
第5页 / 共58页
上海市2013届高三一模考试英语汇编 阅读题 WORD版含答案.doc_第6页
第6页 / 共58页
上海市2013届高三一模考试英语汇编 阅读题 WORD版含答案.doc_第7页
第7页 / 共58页
上海市2013届高三一模考试英语汇编 阅读题 WORD版含答案.doc_第8页
第8页 / 共58页
上海市2013届高三一模考试英语汇编 阅读题 WORD版含答案.doc_第9页
第9页 / 共58页
上海市2013届高三一模考试英语汇编 阅读题 WORD版含答案.doc_第10页
第10页 / 共58页
上海市2013届高三一模考试英语汇编 阅读题 WORD版含答案.doc_第11页
第11页 / 共58页
上海市2013届高三一模考试英语汇编 阅读题 WORD版含答案.doc_第12页
第12页 / 共58页
上海市2013届高三一模考试英语汇编 阅读题 WORD版含答案.doc_第13页
第13页 / 共58页
上海市2013届高三一模考试英语汇编 阅读题 WORD版含答案.doc_第14页
第14页 / 共58页
上海市2013届高三一模考试英语汇编 阅读题 WORD版含答案.doc_第15页
第15页 / 共58页
上海市2013届高三一模考试英语汇编 阅读题 WORD版含答案.doc_第16页
第16页 / 共58页
上海市2013届高三一模考试英语汇编 阅读题 WORD版含答案.doc_第17页
第17页 / 共58页
上海市2013届高三一模考试英语汇编 阅读题 WORD版含答案.doc_第18页
第18页 / 共58页
上海市2013届高三一模考试英语汇编 阅读题 WORD版含答案.doc_第19页
第19页 / 共58页
上海市2013届高三一模考试英语汇编 阅读题 WORD版含答案.doc_第20页
第20页 / 共58页
上海市2013届高三一模考试英语汇编 阅读题 WORD版含答案.doc_第21页
第21页 / 共58页
上海市2013届高三一模考试英语汇编 阅读题 WORD版含答案.doc_第22页
第22页 / 共58页
上海市2013届高三一模考试英语汇编 阅读题 WORD版含答案.doc_第23页
第23页 / 共58页
上海市2013届高三一模考试英语汇编 阅读题 WORD版含答案.doc_第24页
第24页 / 共58页
上海市2013届高三一模考试英语汇编 阅读题 WORD版含答案.doc_第25页
第25页 / 共58页
上海市2013届高三一模考试英语汇编 阅读题 WORD版含答案.doc_第26页
第26页 / 共58页
上海市2013届高三一模考试英语汇编 阅读题 WORD版含答案.doc_第27页
第27页 / 共58页
上海市2013届高三一模考试英语汇编 阅读题 WORD版含答案.doc_第28页
第28页 / 共58页
上海市2013届高三一模考试英语汇编 阅读题 WORD版含答案.doc_第29页
第29页 / 共58页
上海市2013届高三一模考试英语汇编 阅读题 WORD版含答案.doc_第30页
第30页 / 共58页
上海市2013届高三一模考试英语汇编 阅读题 WORD版含答案.doc_第31页
第31页 / 共58页
上海市2013届高三一模考试英语汇编 阅读题 WORD版含答案.doc_第32页
第32页 / 共58页
上海市2013届高三一模考试英语汇编 阅读题 WORD版含答案.doc_第33页
第33页 / 共58页
上海市2013届高三一模考试英语汇编 阅读题 WORD版含答案.doc_第34页
第34页 / 共58页
上海市2013届高三一模考试英语汇编 阅读题 WORD版含答案.doc_第35页
第35页 / 共58页
上海市2013届高三一模考试英语汇编 阅读题 WORD版含答案.doc_第36页
第36页 / 共58页
上海市2013届高三一模考试英语汇编 阅读题 WORD版含答案.doc_第37页
第37页 / 共58页
上海市2013届高三一模考试英语汇编 阅读题 WORD版含答案.doc_第38页
第38页 / 共58页
上海市2013届高三一模考试英语汇编 阅读题 WORD版含答案.doc_第39页
第39页 / 共58页
上海市2013届高三一模考试英语汇编 阅读题 WORD版含答案.doc_第40页
第40页 / 共58页
上海市2013届高三一模考试英语汇编 阅读题 WORD版含答案.doc_第41页
第41页 / 共58页
上海市2013届高三一模考试英语汇编 阅读题 WORD版含答案.doc_第42页
第42页 / 共58页
上海市2013届高三一模考试英语汇编 阅读题 WORD版含答案.doc_第43页
第43页 / 共58页
上海市2013届高三一模考试英语汇编 阅读题 WORD版含答案.doc_第44页
第44页 / 共58页
上海市2013届高三一模考试英语汇编 阅读题 WORD版含答案.doc_第45页
第45页 / 共58页
上海市2013届高三一模考试英语汇编 阅读题 WORD版含答案.doc_第46页
第46页 / 共58页
上海市2013届高三一模考试英语汇编 阅读题 WORD版含答案.doc_第47页
第47页 / 共58页
上海市2013届高三一模考试英语汇编 阅读题 WORD版含答案.doc_第48页
第48页 / 共58页
上海市2013届高三一模考试英语汇编 阅读题 WORD版含答案.doc_第49页
第49页 / 共58页
上海市2013届高三一模考试英语汇编 阅读题 WORD版含答案.doc_第50页
第50页 / 共58页
上海市2013届高三一模考试英语汇编 阅读题 WORD版含答案.doc_第51页
第51页 / 共58页
上海市2013届高三一模考试英语汇编 阅读题 WORD版含答案.doc_第52页
第52页 / 共58页
上海市2013届高三一模考试英语汇编 阅读题 WORD版含答案.doc_第53页
第53页 / 共58页
上海市2013届高三一模考试英语汇编 阅读题 WORD版含答案.doc_第54页
第54页 / 共58页
上海市2013届高三一模考试英语汇编 阅读题 WORD版含答案.doc_第55页
第55页 / 共58页
上海市2013届高三一模考试英语汇编 阅读题 WORD版含答案.doc_第56页
第56页 / 共58页
上海市2013届高三一模考试英语汇编 阅读题 WORD版含答案.doc_第57页
第57页 / 共58页
上海市2013届高三一模考试英语汇编 阅读题 WORD版含答案.doc_第58页
第58页 / 共58页
亲,该文档总共58页,全部预览完了,如果喜欢就下载吧!
资源描述

1、2013届(2012学年)上海市高三英语一模阅读(长宁)(A)When I was in college, a man named Henry worked as a custodian (看门人) in our student union building. White-haired, with a Pennsylvania Dutch accent, Henry could usually be seen in a baseball cap, a T-shirt and a pair of jeans.He was the custodian, and he was the most re

2、spected and most well-known person in the building. Everyone loved Henry, and it was because of all the implicit ways he expressed his love for everyone around him. Henry didnt have to say, “I love you.” He lived his love.Henry was always excited when he met someone new, and he wanted to know everyt

3、hing about them. He felt it was important to do things for people he valued. And Henry seemed to value everyone he met. He brought in articles or cartoons for certain people, went out of his way to introduce people to each other, kept dozens of names and birthdays in his wallet so he could send card

4、s, and helped students keep in touch with graduates who had written to him. He even assisted students who didnt have enough money to buy their books. Henry taught me - and many others he supported - how to live life to the fullest. Not by skydiving or exploring some foreign countries, but by appreci

5、ating where you are in life and valuing those around you.The funny thing is that despite all he did and taught us, Henry truly believed that he was the lucky one - that he was the one who was gaining so much by getting to know us. But all of us who remember Henry know that we were the ones who were

6、truly blessed. We will never forget the man who taught us that best way to say “I love you” often has little to do with the words. 65. What does the word “implicit” in Paragraph 2 most probably mean?A. Unstated.B. Appropriate.C. Indelicate.D. Charming.66. According to the passage, working as a custo

7、dian in a college, Henry _. A. made international travels to live a full lifeB. was a role model of how to be a caring personC. was busy organizing former students reunionsD. gathered plenty information about the needy students67. Henry was always excited to meet new people because _.A. it would mak

8、e people think highly of himB. he was the most popular person in the collegeC. it was part of his responsibility as a custodianD. he treasured everything around him in his life68. Which of the following word cannot be used to describe Henry?A. Gracious.B. Adventurous.C. Supportive.D. Sympathetic.(B)

9、File Created Page 1 of 1 DELAYED BAGGAGE REPORT PRINTED AT ALBANY N.Y. AIRPORT ON JULY 10 2012 11:15 A.M._Dear Valued Customer, We regret that your baggage was not available to claim after your recent flight. Everything possible will be done to locate your property and return it to you promptly. For

10、 information regarding your delayed baggage, contact the United Airlines Baggage Resolution Center at its 24 hour, seven day a week number:1-800-335-BAGS (2247) (US Toll Free)281-821-3536 (Local Houston Number)Or visit site: As soon as you file your Delayed Baggage Report, United Airlines will begin

11、 to trace for your baggage system wide. Our Baggage Resolution staff will make every effort to call you once a day to keep you updated on our progress. So that we can quickly access your records, please refer to the File Reference Number on this receipt when corresponding or calling. Keep this recei

12、pt with your claim check and E-Ticket receipt until your baggage is returned to you. In most circumstances, United Airlines will deliver your baggage when it is located. Delivery times vary depending on location. If your baggage has not been returned to you within the initial five-day tracing period

13、, please download a claim form at and return it to us with the required documentation included._DELAYED BAGGAGE REPORT FILE REFERENCE: ALBUA25876_Name: JOHN JACKSON Contact Number: 802-247-9999Delivery Address: 66 MOONBROOK DRBRANDON Email: minminvtVERMONT USA 05745 _Bag Tag Description0037387643 So

14、ft-Side Upright suitcase0037387657 Non-Zippered, hard side horizontal suitcase69. Where does the receipt most probably come from?A. Suitcase Tracing Website.B. Delivery Center of United Airlines.C. Baggage Resolution Center.D. Information Center of the Airport.70. From the receipt we can learn that

15、John Jackson _.A. must have filed his Delayed Baggage Report alreadyB. couldnt deal with his delayed baggage on line until July 17, 2012C. should call 802-247-9999 for the information about his delayed baggageD. may not know anything about his delayed baggage during the first five days 71. Which of

16、the following information is not provided in the receipt? A. The description of the customers checked suitcases.B. The regular routine of the delayed baggage tracing.C. The contact phone number of Albany N.Y. Airport. D. The customers delayed baggage file reference number. (C)The candidate longing f

17、or election to the highest office in the United States must be native-born American citizen who is at least 35 years old and who has lived in the United States for at least 14 years. The election course is complicated, and the road to the Oval Office is long.Potential candidates must present papers

18、stating their intention to seek their partys recommendation; delegates then choose from among those running some months later at the national conferences. Before that, however, each state holds a primary election that determines how the states voters want the partys delegates to vote. Methods of cho

19、osing the delegates vary from state to state. At the conferences, there are speeches and often heated discussions. It may take several rounds of voting before delegates can agree on a candidate. On the final day of the meeting, the presidential candidate announces his or her choice for vice presiden

20、t. Election Day, by law, is the Tuesday that follows the first Monday in November. On this day, registered voters may cast their votes for president, vice president, and candidates for other federal (联邦), state, and local offices. The popular vote, however, does not determine who will be president.

21、The president is chosen by a vote of the Electoral College, a group of 538 citizens from the 50 states and the District of Columbia who are chosen to cast votes for the president and vice president. The rules for choosing electors, as with the delegates, vary. Each state also decides whether its ele

22、ctoral votes must reflect the popular vote. The number of electors in each state is determined by the number of representatives and senators (议员) that a state sends to Congress and, therefore, may change every 10 years, depending on the results of the United States census (人口普查). The winner must get

23、 at least 270 of the electoral votes when the Electoral College meets in December of the election year. The election results are not official, however, until the following January, when Congress meets in a joint conference to count the electoral votes.At the end of the road to the Oval Office is the

24、 swearing-in ceremony, at which the new or reelected president takes the oath (誓言) of office on January 20. 72. Which of the following is not a requirement for candidates for the presidency of the USA?A. Minimum age of thirty-five years old.B. Previous experience in an elective office.C. Native of t

25、he United States of America.D. Bottom residency of fourteen years in the States.73. Which of the following statements is true according to the passage?A. Election results are not authorized until the first month of the coming year.B. Ways to select the delegates of each state are very much the same

26、in the USA.C. The successful presidential candidate wins a maximum number of electoral votes.D. A partys potential candidate is chosen months before its national conference is held.74. We can conclude from the passage that _. A. the candidate who wins the popular vote will be the winner of the elect

27、ionB. the electors dont have the right to vote against their registered voters willC. the presidents of the United States are theoretically picked by the citizens directlyD. the change of a states population barely has effect on the presidential election75. Which of the following might be the best t

28、itle of the passage? A. The Various Ways of Presidential Election in the USAB. The Qualifications for Being the President of the USAC. The Rules for Choosing Presidential Candidates in the USA D. The Process of the Election for the President of the USA6568ABDB6971CAC7275BACD(闸北)(A)Mike frowned (皱眉)a

29、t his calendar and then picked up his guitar. His band had been together for five months. Everyone said they were fantastic but they still had not been blessed with a paying job. The three guys had become acquainted in band class, and organizing a group had been Mikes idea. Mike played the guitar an

30、d acted as leader. john was on drums and James was on the saxophone. The three got together just for fun and named their group Playday. But when some fans praised their talent, they decided to turn fun into profit.To make themselves known, Playday volunteered to entertain at parties. They played for

31、 free at school get-togethers. Mike loved the applause! He felt like a superstar.They practiced hard and discussed young musicians who had vaultea into prominence. “The Jackson Five were kids, John said. “Michael shot to fame like a rocket! He exclaimed. “We can do it! Mike encouraged the group.Afte

32、r months of keeping Spirits high, Mike was tired. They havent seen even a cent. Let alone making a killing. Everyone was in low spirits. They decided to break up after one final volunteer job. The next day, they were to perform for patients at the Childrens Hospital where Mikes mother worked as a nu

33、rse. The next day the three young guys set up in the hospital lobby, and the nurses brought in the audience. As Playday began its number, Mike could feel their lack of spirit.Then Mike spotted one patient sitting limply in her wheelchair. Her expression was vacant, but when music filled the room, th

34、is rag doll came to life. She sat up straight, and the light that shone in her eyes lighted up something in Mike. He felt the thrill hed been missing. As Mikes playing caught fire, James and John gained energy. Soon Playday was rocking as never before.After the performance, the three guys spent time

35、 with the patients. Mike spoke to the girl who had encouraged him to play so well. “Wouldnt it feel great to be rich and famous? she asked, eyes still bright. “Will you be a rock star?“Probably not, Mike answered. “But Ill never give up my music. It felt like the big time playing for you!25. The und

36、eRLiNed phrase “had vaulted into prominence” most likely means . A. had grown into adultsB. had made a promise C. had played The Jackson FiveD. had bEcome successful25. The members of Playboy were tired and depressed because they.A. had no offer of a paying performance B. competed cruelly with other

37、 bands C. called an end to the bandD. performed at the Childrens Hospital25. While performing at the Childrens Hospital, Mike became inspired because.A. his guitar caught fireB. a girl took him as a rock starC. a patient was much cheered upD. a toy doll suddenly became alive25. Which of the followin

38、g statements is TRUE according to the passage?25. MiKe, John and James got to know each other as they were relatives.25. They planned to make money when they had a big fan base. 25. Their wonderful performance at Childrens Hospital cured the girl of her disease.25. Mike found out the enjoyment of vo

39、lunteer performance after playing at the hospital.(B)Read this tourist booklet for Clarke Qiuay:Souvenirs from the PastEvery weekend, theres a popular flea (跳蚤)market. Hunt for treasures of a different kind. Among a collection of goodies, youll fiNd jewelries, antiQUES ANd carpets that are centuries

40、 old, which deFiNES Their significance. All THe specialty shops here deal in ancient items, including remains of the past.Dine by the RiverThe high-tech centers which tower over the historic riverside buildings bring a modern taste to Clarke QuAY. When it comes to food, you are spoilt for a variety

41、of choice. Sample local favorites in the cool comfort of the food court or enjoy them in the open at a snack stand. You may also experience special spirits at any of the watering holes.Nightlife EntertainmentClarke Quay boasts a bustling nightlife. Magic and music fill the air. Trolleys on wheels bu

42、rst along streets selling sweets of unique shapes and colors. Fortune tellers cast their spell and told forbidden stories. The atmosphere is boiling.The riverside village plays host to a good number of watering holes, all of which feature nightly live entertainment. Sit back with a drink there and w

43、atch the local colour while your favourite music washes over you. Or you may hit the dance floor and flash your moves.A Ride into the Past and the FutureYou may choose to arrive by means of a vessel from the past. The unique river taxi was previously a boat that transported goods from ship to land.

44、These days, it dominates the river, transporting tourists and locals to their various destinations. How to get here:25. From City Hall MRT Station:Take Bus Service 32/135 along North Bridge Rd.26. From Orchard MRT Station: Take Bus Service 54 along Scotts Rd.26. The gifts visitors can buy from the f

45、lea market are mainly.A. carpets produced in another country B. animals raised in the marketC. second-hand objects of historlcal value D. candies of different shapes and colors26. In the booklet the underlined watering holes” are closet in meaning to .A. mental hospitalsB. night clubsC. CD shopsD. e

46、ntertainment shows26. According to the booklet, which of the following statements is NOT TRUE?26. Different kinds of food are offered in Clarke Quay.26. Fortune telling is a forbidden business in Clarke Quay.26. River taxis ship people instead of goods today.26. The bus service 54 can bring visitors

47、 to Clarke Quay from Orchard MRT Station.(C)Announcing recent he would send proposals on reducing gun violence in America, President Obama mentioned a number of sensible gun-control measures. But he also paid homage to the Washington conventional wisdom about the many and varied causes of shooting c

48、ases. He said earlier that gun violence was a complex problem that will require a complex solution and gun control, therefore, was far from the only answer. Then are the data shielding the politicians vague language?America is a gun heaven. Around 11,000 deaths were caused by guns last year. In cont

49、rast, Britain has about 50 gun killings a year. Many people believe that America is simply a more violent, individualistic society. But the only field in which the U.S. rate is surprisingly higher seems to be the gun killings. For all the other crimes theft, robbery, attack the United States is with

50、in the range of other advanced countries. Is Americas popular Culture the cause? This is highly unlikely, as largely the same culture exists in other rich countries. Youth in Britain, for example, are exposed to almost the same cultural influences as in the United States. The Japanese are at the cut

51、ting edge of the world of video games, most of which touch on violence. Yet the rates of gun shooting in these two countries are a tiny function of Americas. At the same time, Britain and Japan both have perhaps the tightest regulations of gun. As for America, the country has far more permissive gun

52、 laws. With 5 percent of the worlds population, the United States has 50 percent of the guns.There are always evil or weak-minded people, who might be influenced by popular culture. But how can government identify the darkest thoughts in peoples minds before they have taken any action? Certainly tho

53、se who urge all-round democracy would not allow government to monitor thoughts, forbid free expression, and ban the sale of information and entertainment in exchange for bodily safety. Then why not do something much simpler and that has been successful: limit access to guns? America is in desperate

54、need of a real ban, not another toothless ban, full of exceptions, which the gun lovers would use to claim that such bans dont reduce violence.26. In the first paragraph President Obama.26. regarded loose gun control as the single reason for gun violence26. thought many reasons accounted for the gun

55、 violence in America 26. believed Americas gun laws had nothing to do with gun violence26. thought gun violence was far from the only problem America faced26. Cultural influence is not the main reason for gun violence because.26. Americans are not influenced by violent video games as Japanese are26.

56、 cultural influences usually cause theft or robbery rather than gun firing26. nations of similar cultural background have lower rates of gun shooting than the U.S.26. Americans are rarely influenced by popular culture 26. In the passage, the author mainly discusses.26. President Obamas lack of coura

57、ge and poor leadership26. the crime rates of different countries of similar cultural background26. the main cause of gun violence in American and the solution26. the comparison between losing democracy and bodily safety26. According to the passage, which of the following statements is TRUE?26. Washi

58、ngton conventional wisdom agrees on strict gun control.26. Identifying evil people in advance can be done with the help of democracy.26. Controlling access to guns is a good way to solve the gun shooting problem.26. Some exceptions should be allowed for the gun bans. 6568 DACD 6971 CBB 7275 BCCC(杨浦)

59、(A)Brigham Young University students can now receive the unconditional love of dogs without breaking rules prohibiting pets in university housing. Jenna Miller started her company Puppies for Rent this summer in the Provo area as a way for students and others to rent puppies by the hour. The pups ha

60、ve been rented for first dates and surprise parties and by mothers rewarding their children. After signing a contract, customers can rent them for $15 an hour, $25 for two hours and $10 for each additional hour. Miller offers her seven puppies for playtime rentals, with each dog hand delivered strai

61、ght to the customers door. Her lawyer brother helps her with legal contracts and fees. She now has four employees helping look after and deliver the animal. nova Carl Arky, spokesman for the Humane Society of Utah said his group is against the business. Puppies need consistency and stability in thei

62、r lives, he said, and renting them to various people might affect the animals growth and development. Miller said the animals are treated well and she has a 100 percent success rate so far finding them a permanent home. Money paid by renters goes toward adoption fees if they decide to own a puppy.65

63、. Which of the following are not possible renters of the puppies? A. Young lovers. B. Party organizers. fuck myself C. Mothers. D. Scientific group members. 66. Millers brothers main responsibility is to _. A. draft contracts pig B. deliver animals C. find adoption families D. walk dogs 67. Why is C

64、arl in disagreement with the service? A. Because playing with pets is harmful to childrens health. B. Because some people will be cruel to the rented animals. aeroplane C. Because unstable living environment is not good for animals growth. D. Because it will prohibit the puppies from finding a perma

65、nent home. (B)TENANCY AGREEMENT DEFINITIONS THE LANDLORD Mrs Gloria Black of 6 Sutton Road, Cambridge CB5 7AQ THE TENANT Marina Kahn is it? PROPERTY 24a Wood Road, Cambridge CB2 8BG TOGETHER WITH CONTENTS (fixtures, furniture and equipment) specified in the inventory (attached) TERM from 1 st Januar

66、y 20 _ to 31 st December 20 _ (12 months) RENT 500 per calendar month, payable in advance on the first day of each month DEPOSIT 500, payable on commencement of this Agreement lets AGREEMENTS A The Landlord may re-enter the Property and terminate this Agreement if the Rent or any part of it is not p

67、aid within fourteen days after it becomes due. B The Landlord may bring the tenancy to an end at any time before the expiry of the Term (but not earlier than six months from the Commencement Date of this Contract) by giving the Tenant not less than two months written notice starting that the Landlor

68、d requires possession of the Property. pilot C The Landlord shall put the deposit with the Deposit Protection Service, and shall inform the Tenant within 14 days of taking the deposit of the contact details of this service and details of how to apply for the release of the deposit from this service.

69、 fuck myself TENANTS OBLIGATIONS 1 Pay the Rent into the Landlords bank account at the times specified. 2 Pay for all water, gas and electricity consumed on the Property during the Term; and pay in full for all charges made for the use of telephone on the Property during the Term. 3 Keep the interio

70、r of the Property during the Term in a good and clean state of repair, condition and decoration. 4 Permit the Landlord to enter the Property at all reasonable times; to inspect the Property and its contents; and to carry out any works of maintenance or repair to the Property; to show prospective new

71、 Tenants around the Property at the end of the tenancy. 5 Not take in any paying guest without the prior written consent of the Landlord. 6 Not use the Property other than as a private dwelling; nor carry on any profession, trade or business in the Property. 7 Not use any musical instrument, wireles

72、s or television between midnight and 7 am, nor permit any singing or dancing between these hours. 8 Not keep in the Property any cat, dog or other pet without the prior written consent of the Landlord. SIGNATURES Tenant: Name (print): MARINA KAHN Signed: Marina Kahn Witnessed by: Name (print): ALICE

73、 RACE Signed: A. RACE Occupation: SECONDARY SCHOOL TEACHER Address: 16 School House Lane, Cambridge, CB2 8GH Tenant: Name (print): GLORIA BLACK Signed: Gloria Black Witnessed by: Name (print): OLIVE LYDIA LEE Signed: Olive L Lee Occupation: Librarian Address: 3 Fieldhead Road, Peterborough PB4 8DU68

74、. Whats the monthly rent of the property? A. 500. B. 575. C. 1000. D. 1500. 69. Whats the landlords witness? A. A teacher. B. A librarian. C. A house agent. D. A bank clerk. 70. Which of the following is allowed in the property? A. Watching TV at any time. B. Holding an all-night dancing party. C. C

75、hanging it into a business office. D. Entertaining friends with self-cooked meals. 71. Which of the following is the right of the landlord? A. He can show new tenants around the property at any time. B. He can enter the property to inspect its contents. C. He can take back his property whenever he w

76、ants. D. He can keep the deposit for himself. (C)Does solving a math problem give you a headache? Do you feel nervous when you sit a math exam? For most students, math can be tough but scientists have proved that math problems can actually trigger physical pain. Scientists came to his conclusion wit

77、h an in-depth experiment, which was published in the Public Library of Science One journal. They began by finding out how much participants fear math. Those involved were asked a series of questions such as how they feel when they receive a math textbook or when they walk into a math lesson. Based o

78、n their answers, participants were divided into groups. One group was made up of people who were particularly afraid of math and participants in the other group were more comfortable with the subject. Both groups were then given either math tasks or word tasks. When a math task was going to come nex

79、t, a yellow circle would appear but when a word task was soon to come, a blue square would be shown. Using a brain-scan machine, scientists noticed that whenever people from Group One saw a yellow circle, their brain would respond in a way similar to when their body is feeling pain. It was like the

80、pain they would fee, for example, if they burnt their hand on a hot stove. But they reacted less strongly when they knew that they would be faced with a word task. “ However, scientists saw no strong brain response from people in the second group. Math can be difficult, and for those with high level

81、s of mathematics-anxiety (HMA), math is associated with tension, apprehension and fear. “When you are really thinking about the math problems, your mind is racing and you are worrying about all the things that could go wrong,” explained Ian Lyons from University of Chicago, US, leader of the study.

82、“The higher a persons anxiety of a maths task, the more he activated brain regions associated with threat detection, and the experience of pain.”More interestingly, the brain activity disappeared when participants actually started dealing with the math tasks. “This means that its not that math itsel

83、f hurts; rather, the anticipation of math is painful,” Lyons said. Based on the study, scientists suggested that things could be done to help students worry less and move past their fear of math, which might mean they perform better in tests. 72. In the first stage, scientists ask participants some

84、questions to _. A. see whether math hurts B. find out how much they fear math C. observe how their brain response D. test their endurance of pain 73. The underlined word “the anicipation of math” is closest in meaning to _. A. the attempt of learning math B. the motivation to work out math problem C

85、. the effort to understand math D. the act of thinking about math 74. Which is the best title for the passage? A. How to overcome math fear. B. Physical pain affects math performance. C. Math pain in your brain. D. Unknown truth about pain. 75. What can be concluded from the experiment? A. The antic

86、ipation of math has no relation to students confidence in math. B. Moderate mathematic anxiety promotes students academic performance. C. Effective solutions have been worked out to lower students anxiety of math. D. Physical pain caused by HMA disappears in the process of doing math problem. 6567 D

87、AC 6871 ABDC 7275 BDCD (徐汇)(A)Marlon Brando is widely considered the greatest movie actor of all time.Hewas born in Omaha Nebraska in 1924. He was named after his father, a salesman. His mother, Dorothy, was an actress in the local theater.Marlon Brando moved to New York City when he was 19 years ol

88、d in 1943. He took acting classes at the New School for Social Research. One of his teachers was Stella Adler, who taught the Method style of realistic acting. The Method teaches actors how to use their own memories and emotions to identify with the characters they are playing.Marlon Brando learned

89、the Method style quickly and easily. Critics said he was probably the greatest Method actor ever. One famous actress commented on his natural ability for it. She said teaching Marlon Brando the Method was like sending a tiger to jungle school.Marlon Brando appeared in several plays. He got his first

90、 major part in a Broadway play in 1947, at the age of 23. He received great praise for his powerful performance at Stanley Kowalski in Tennessee Williams play,A Streetcar Named Desire. His fame grew when he acted the same part in the movie version, released in 1951.Marlon Brando won the Best Actor O

91、scar forThe God Father, but he rejected it. He sent a woman named Sasheen Littlefeather to speak for him at the Academy Awards ceremony. She said that Brando could not accept the award because of the way the American film industry treated Native Americans. The people at the Academy Awards ceremony d

92、id not like the speech. But some experts think the action helped change the way American Indians were shown in movies.Marlon Brando acted in about forty movies. He was nominated for a total of eight Academy Awards. But he earned a bad boy reputation for his public outbursts and unusual behaviours. A

93、ccording toLos Angelesmagazine, Brando was rock and roll before anybody knew what rock and roll was. His later life was marked with family tragedies. His son Christian went to prison for killing his daughter Cheyenes boyfriend. Cheyene later committed suicide. Brando became lonely. He worked occasio

94、nally for the money. But, in his prime, Marlon Brando was an actor other actors could only hope to become.65.According to the passage, the Method _.A. requires actors to use their imagination in actingB. made Marlon Brando a great and famous actorC. wasnt very difficult for Marlon BrandoD. can be mo

95、st effectively learned in a jungle66.The speech at the Academy Awards ceremony _ at the time.A. made Sasheen Littlefeather well-knownB. was well-received by Native AmericansC. changed peoples attitude to American IndiansD. received both positive and negative responses67.Which of the following is NOT

96、 true according to the passage?A. Marlon Brandos later life was troubled and unhappy.B.A Streetcar Named Desirewas later adapted into a movie.C. Marlon Brando was the first rock star in USA.D. As an actor, Brandos talent was unparalleled.68. The purpose of this passage is to _.A. inform us of Marlon

97、 Brandos attitude to civil rights movementB. introduce Marlon Brando as one of the greatest actorsC. help us understand Marlon Brandos secret to successD. instruct us how to become a great actor like Marlon Brando(B)Learning English Video Project Encounters in the UK(17 minutes)Watch|CommentsEncount

98、ers in the UKis the first film in this documentary mini-series. It tells the story of four girls from different countries who travelto Cambridge in England to study English and stay with local families in what is called a homestay arrangement. For the four girlsthe homestay arrangementis a positive

99、experience. As one of the homestay hosts explains: Its going to be a great experience, not only in terms of learning English, but in learning about life.Watch with:subtitles|transcript|no subtitles|Comments2. Stories from Morocco(16 minutes)Watch|Comments in Casablanca, Morocco, this film features f

100、ootage and interviews focusing on key questions such as Why are people learning English? and What tips and advice can learners offer? Staff and learners discuss theadvantages and challenges of English language learning in Morocco. Interviewees touch on a variety of topics including British vs. Ameri

101、can accents, multi-level classrooms, and the similarities of English to French and Spanish.Watch with Subtitles|Watch without Subtitles|Comments3.Thoughts from Brazil(17 minutes)Watch|CommentsLikeInsights from China, Thoughts from Brazilalso looks at modern trends in learning English, especially for

102、 children and teens. It will be of particular interest to all those who long for a learning experience that is more interactive and communicative. Teens and young adults will find new ideas for combiningpersonal interests such as music, gaming and social media with self-study. As Daniel Emmerson tal

103、ks to learners and teachers of English in Sao Paulo, Brazil, he discovers that many of them have found for themselves the principle of learning by doing and have readily adapted it to the Internet era.Watch with Subtitles|Watch without Subtitles|Comments69. From the passage we can conclude that “Lea

104、rning English Video Project” is most probably _.A. an online language learning courseB. audio documents on language learningC. a series of short video programsD. a set of films on English-speaking countries70. If someone is interested in the comparison between English and other languages, he might b

105、e interested to watch _.A. Encounters in the UK B.Stories from MoroccoC.Thoughts from BrazilD.Insights from China71. What can we know about English learningin Sao Paulo, Brazil?A. Classroom teaching is more interactive and communicative.B. Homestay arrangement provides positive experience for learne

106、rs.C. The Internet and games plays a major role in language learning.D. The principle of learning by doing is widely accepted by learners.(C)It is found that American students spend less than 15% of their time in school. While theres no doubt that school is important, a number of recent studies remi

107、nds us that parents are even more so. A study published earlier this month by researchers at North Carolina State University, for example, finds that parental involvement checking homework, attending school meetings and events, discussing school activities at home has a more powerful influence on st

108、udents academic performance than anything about the school the students attend. Another study, published in the Review of Economics and Statistics, reports that the effort put forth by parents (reading stories aloud, meeting with teachers) has a bigger impact on their childrens educational achieveme

109、nt than the effort devoted by either teachers or the students themselves. And a third study concludes that schools would have to increase their spending by more than $1,000 per pupil in order to achieve the same results that are gained with parental involvement.So parents matter. But it is also reve

110、aled in researches that parents, of all backgrounds, dont need to buy expensive educational toys or digital devices for their kids in order to give them an advantage. They dont need to drive their offspring to enrichment classes or test-preparation courses. What they need to do with their children i

111、s much simpler: talk.But not just any talk. Recent research has indicated exactly what kinds of talk at home encourage childrens success at school. For example, a study conducted by researchers at the UCLA School of Public Health and published in the journal Pediatrics found that two-way adult-child

112、 conversations were six times aspotentin promoting language development as the ones in which the adult did all the talking. Engaging in thisreciprocal(双向的) back-and-forth gives children a chance to try out language for themselves, and also gives them the sense that their thoughts and opinions matter

113、.The content of parents conversations with kids matters, too. Children who hear talk about counting and numbers at home start school with much more extensive mathematical knowledge, report researchers from the University of Chicago. While the conversations parents have with their children change as

114、kids grow older, the effect of these exchanges on academic achievement remains strong. Research finds that parents play an important role in what is called “academic socialization” setting expectations and making connections between current behavior and future goals. Engaging in these sorts of conve

115、rsations has a greater impact on educational accomplishment.72.Parents are even more important than schools in that _.A. parental involvement makes up for what schools are not able to doB. teachers and students themselves do not put in enough effortC. parental involvement saves money for schools and

116、 the local governmentD. students may well make greater achievements with parents attention73.It can be inferred from the 2ndparagraph that _.A. educational toys are unaffordable nowadaysB. digital devices can give children an advantageC. some parents believe in enrichment classesD. talking with chil

117、dren is a very simple task74.The word potent is closest in meaning to _.A. powerfulB. difficultC. necessaryD. resistant75.Which of the following will more encourage childrens success at school according to the passage?A. Parents order their children to stop playing video games.B. Parents discuss wit

118、h their children the possible future career.C. Parents lecture their children on getting too low marks on tests.D. Parents introduce colleges around the US to their children.65-68CDCB69-71CBD72-75 DCAB(松江)FROM THE CHINESE PRESS (Grandmas not-so-happy tale)An old villager traveled all the way from Hu

119、bei province to Nanjing, Jiangsu Province to see his grandchild. But his son, instead of letting him spend time with his grandchild, put him up in hotel. The old man felt humiliated because he thought his son had treated him indifferently because he was poor. The son should definitely be criticized

120、for his action, but at the same time it is important to analyze the incident rationally, says an article in Qilu Evening News. Excerpts: The case of the old man from Hubei is another tragic Chinese family drama. Chinese tradition does not allow a son to shut out his father from His family. But conde

121、mning the sons action without knowing the details may not be a good idea. It indeed was inappropriate for the son not to take his father to his house. But it is equally inappropriate to accuse him of being heartless for picking up his father at the railway station and putting him up in a hotel. It i

122、s possible that the son had private reasons for not taking his father home. Perhaps he didnt want to burden his father with additional worries that he could have felt had he gone to his house.We should not jump to conclusions without knowing why the son didnt take the old man to his house. It is not

123、 rational to say that the son is ashamed of his father because he is a poor rural resident. When facts are not clear, making efforts to find them is more important than leveling accusations against a person. 65. Which of the following words can be used to replace the underlined word “rational” in Pa

124、ragraph 3? A. Sensible. B. Unreasonable. C. Reachable. D. Sensitive.66. Which of the following is correct according to the passage?A. The old villager traveled all the way to borrow money from his son.B. The son should be accused of being heartless whatever the reason is.C. It is not proper for the

125、son not to take his father to his house even if there is some reason.D. The son is ashamed of his father just because he is a poor rural resident.67. According to the writers analysis, we should _.A. help the son to explain the reason to the publicB. find out the reason why the son treated his fathe

126、r like thatC. criticize the son for his action D. draw the conclusion that the sons behavior is against Chinese tradition68. The writers attitude toward the sons way of treating his father is _.A. critical B. favorable C. objective D. negative(B)Events Calendar TUESDAYLandscape Pests(虫子)Learn to ide

127、ntify, control and prevent seasonal landscape-disease and landscape-pest problems at the workshop, 3:30 pm-5 pm. Tuesday at the US National Arboretum,3501 New York Ave NE, Washington, $15; registration required.202-245-4521 orwww.usna.usda.gov.THROUGH AUG.3Horticultural(园艺的)ArtWatercolors, pen-andin

128、k drawings and colored-pencil pieces by the Brookside Gardens Schoo1 of Botanical Art and Illustration will be on display at the exhibit Botanica 2007: the Art and Science of Plants at Brookside Gardens Visitors Center, 1800 Glena11an Ave, Wheaton, through Aug. 3. Free. 301-962-1400 orwww.brooksideg

129、ardens.org.THROUGH SEPT.9Botanical(植物学的)ArtVisit Patterns in Nature, an exhibit by Amy Lamb featuring photographs of flowers, leaves and other botanical life, at the US Botanic Garden Conservatory, West Orangery, 100 Maryland Ave SW, Washington, through Sept. 9. The conservatory is open 10 am-5 pm d

130、aily. Free. 202-225-8333.THROUGH OCT.8Botanic Garden ExhibitCelebrating Americas Public Gardens is on view through Oct. 8 at the US botanic Garden, 100 Maryland Ave SW, Washington. The exhibit, on the Conservatory Terrace and in the National Garden, features displays of 20 public botanic gardens acr

131、oss the country. Hours are 10 am-5 pm daily. Free. 202-200-8956.68. If you want to record your name for an event in advance, you may call .A. 202-225-8333 B. 202-245-4521C. 301-962-1400 D. 202-200-895669. If you go to Botanic Garden Exhibit, you .A. can enjoy drawings and colored-pencil piecesB. can

132、 learn how to kill pests living on the plantsC. can find displays of 20 botanic gardens across the countryD. will enjoy the photographs of flowers and leaves70. From the advertisement, we learn .A. the first event is about growing healthy plantsB. all of the events are free of chargeC. there is no t

133、ime limit to all the eventsD. you can find all the information of the events either by phone or e-mail (C)The new year has come and it is time for those about to graduate to look for jobs. Competition is tough, so job seekers must carefully consider their personal choices. Whatever we are wearing, o

134、ur family and friends may accept us, but the workplace may not.A high school newspaper editor said it is unfair for companies to discourage visible tattoos (纹身)nose rings, or certain dress styles. It is true you cant judge a book by its cover, yet people do“cover”themselves in order to convey (传递)ce

135、rtain messages. What we wear, including tattoos and nose rings, is an expression of who we are. Just as people convey messages about themselves with their appearances so do companies. Dress standards exist in the business world for a number of reasons, but the main concern is often about what custom

136、ers accept.Others may say how to dress is a matter of personal freedom, but for businesses it is more about whether to make or lose money. Most employers do care about the personal appearances of their employees (雇员), because those people represent the companies to their customers.As a hiring manage

137、r I am paid to choose the people who would make the best impression on our customers. There are plenty of well-qualified candidates, so it is not wrong to reject someone who might disappoint my customers. Even though I am open-minded, I cant expect all our customers are.There is nobody to blame but

138、yourself if your set of choices does not match that of your preferred employer. No company should have to change to satisfy a candidate simply cause he or she is unwilling to respect its standards, as long as its standards are legal.70. The authors attitude towards strange dress styles in the workpl

139、ace may best be described as_.A. enthusiasticB. negativeC positiveD. sympathetic71. Which of the following is the newspaper editor opinion according to Paragraph 2?A. Strange dress styles should not be encouraged in the workplace.B. Candidates are supposed to wear whatever they would love to.C. Cand

140、idates with tattoos or nose rings should be fairly treated. D. Peoples carrying tattoos, nose rings are regarded as guilty.72. What can be inferred from the text?A. Hiring managers try to make the best impression of themselves on their candidates.B. What to wear is a matter of personal choice for co

141、mpanies.C. Companies sometimes have to change to respect their candidates.D. Candidates have to wear what companies prefer for an interview.73. Which of the following would be the best title for the text?A. Reap What You Sow B. Think Before You Leap C. Repair The House Before It RainsD. Give The Mon

142、key What He Wants6568 ACBC6971 BCD 7275 BADD(青浦)(A)On Thursday afternoon Mrs. Clarke, dressed for going out, took her handbag with her money and her key in it, pulled the door behind her to lock it and went to the over 60s Club. She always went there on Thursdays. It was a nice outing for an old wom

143、an who lived alone.At six oclock she came home, let herself in and at once smelt cigarette smoke. Cigarette smoke in her house? How? How? Had someone got in? She checked the back door and the windows. All were locked or fastened, as usual. There was no sign of forced entry.Over a cup of tea she wond

144、ered whether someone might have a key that fitted her front door“a master key” perhaps. So she stayed at home the following Thursday. Nothing happened. Was anyone watching her movements? On the Thursday after that she went out at her usual time, dressed as usual, but she didnt go to the club. Instea

145、d she took a short cut home again, letting herself in through her garden and the back door. She settled down to wait.It was just after four oclock when the front door bell rang. Mrs. Clarke was making a cup of tea at the time. The bell rang again, and then she heard her letter-box being pushed open.

146、 With the kettle of boiling water in her hand, she moved quietly towards the front door. A long piece of wire appeared through the letter-box, and then a hand. The wire turned and caught around the handle on the door-lock. Mrs. Clarke raised the kettle and poured the water over the hand. There was a

147、 shout outside, and the skin seemed to drop off the fingers like a glove. The wire fell to the floor, the hand was pulled back, and Mrs. Clarke heard the sound of running feet.65. Mrs. Clarke looked forward to Thursday because.A. she worked at a club on the dayB. she had visitors on ThursdayC. she v

148、isited a club on ThursdayD. a special visitor came on Thursday66.What does the expression “a master key” in Paragraph 3 most probably mean?A. A key to all the doors.B. A key only to the womans door.C. A key only its master can use.D. A key not everyone can use.67. On the third Thursday Mrs. Clarke w

149、ent out.A. because she didnt want to miss the club againB. to see if the thief was wandering outsideC. to the club but then changed her mindD. in an attempt to trick the thief68. The lock on the front door was one which.A. needed a piece of wire to open itB. could be opened from inside without a key

150、C. couldnt be opened without a keyD. used a handle instead of a key(B)RichardSolo 1800 Rechargeable BatteryIn just minutes a day, plug in and charge your iPhone quickly!Just plug RichardSolo 1800 into your iPhone once or twice a day, for fifteen minutes, and keep your iPhone charged up. At your desk

151、, or at dinner, plug RichardSolo into iPhone to instantly transfer charge. No more battery worries. RichardSolo will charge iPhone to full 1.5 times, and it is good for 3-5 years of recharges.Use the iPhone while charging it. Even charge the RichardSolo 1800 and iPhone together at the same time. Tak

152、e only one charger when traveling and wake up in the morning with the RichardSolo and the iPhone charged.RichardSolo 1800 is largest in its class and holds its charge for months. Works with almost all iPhone cases.Your satisfaction is guaranteed, with our 30-day return privilege. If youre not satisf

153、ied for any reason, well email you a pre-paid return label.Actual customer comments:To have your company exhibit such good service is unbelievably refreshing. P.S.This is what I call great customer support. I wish more companies would figure this out these days. Thank you so much. D.C.You have provi

154、ded me one of the best services I have ever seen on any online/ telephone shopping. T.K.You must have the fastest processing and shipping in the industry! M.C.This is the best customer service experience I have had in a long time. If only we always had such good enjoyment! L.L.Ive read online about

155、your amazing customer service, and I must say Im now a true believer. B.L.Your service is so satisfactory that I even cant believe it, and I will introduce it to my friends.J.K.69. What is special about the battery?A. It is built in an iPhone.B. It is the smallest of its kind.C. It can also be used

156、as a charger.D. It keeps power for about 30 days.70. Who mentions the transporting of the battery?A. P.S.B. B.L.C. M.C.D. T.K.71. The customer comments on the battery are mainly about its.A. qualityB. service C. functionD. shopping(C)At the heart of the debate over illegal immigration lies one key q

157、uestion: are immigrants good or bad for the economy? The American public overwhelmingly thinks theyre bad. Yet the agreement among most economists is that immigration, both legal and illegal, provides a small net boost to the economy. Immigrants provide cheap labor, lower the prices of everything fr

158、om farm produce to new homes, and leave consumers with a little more money in their pockets. So why is there such a discrepancy between the perception of immigrants impact on the economy and the reality?There are a number of familiar theories. Some argue that people are anxious and feel threatened b

159、y an inflow of new workers. Others highlight the stress that undocumented immigrants place on public services, like schools, hospitals, and jails. Still others emphasize the role of race, arguing that foreigners add to the nations fears and insecurities. Theres some truth to all these explanations,

160、but they arent quite sufficient.To get a better understanding of whats going on, consider the way immigrations impact is felt. Though its overall effect may be positive, its costs and benefits are distributed unevenly. David Card, an economist at UC Berkeley, notes that the ones who profit most dire

161、ctly from immigrants low-cost labor are businesses and employersmeatpacking plants in Nebraska, for instance, these producers savings probably translate into lower prices at the grocery store, but how many consumers make that mental connection at the checkout counter? As for the drawbacks of illegal

162、 immigration, these, too, are concentrated. Native low-skilled workers suffer most from the competition of foreign labor. According to a study by George Borjas, a Harvard economist, immigration has reduced the wages of American high-school dropouts by 9%.Among high-skilled, better-educated employees

163、, however, opposition was strongest in states with both high numbers of immigrants and relatively generous social services. What worried them most, in other words, was the financial burden of immigration. That conclusion was reinforced by another finding: that their opposition appeared to soften whe

164、n that financial burden decreased, as occurred with welfare reform in the 1990s, which curbed immigrants access to certain benefits.The irony is that for all the overexcited debate, the net effect of immigration is minimal. Even for those most acutely affectedsay, low-skilled workers, or California

165、residentsthe impact isnt all that dramatic. “The unpleasant voices have tended to dominate our perceptions,” says Daniel Tichenor, a professor at the University of Oregon. “But when all those factors are put together and the economists calculate the numbers, it ends up being a net positive, but a sm

166、all one.” Too bad most people dont realize it.72. What can we learn from the first paragraph?A. Whether immigrants are good or bad for the economy has been puzzling economists.B. The American economy used to thrive on immigration but now its a different story.C. The agreement among economists is tha

167、t immigration should not be encouraged.D. The general public thinks differently from most economists on the impact of immigration.73. What is the chief concern of native high-skilled, better-educated employees about the inflow of immigrants?A. It may change the existing social structure.B. It may po

168、se a threat to their economic status.C. It may decrease .their financial burden.D. It may place a great pressure on the state budget.74. What is the irony about the debate over immigration?A. Even economists cant reach an agreement about its impact.B. Those who are opposed to it turn out to benefit

169、most from it.C. People are making too big a fuss about something of small impact.D. There is no essential difference between seemingly opposite opinions.75.Which of the following might be the best title of the passage?A. A debate about whether to immigrate.B. A debate about the impact of illegal imm

170、igrants. C.The great impact of immigrants on the economy.D. Opposition to illegal immigration.6568CADB 6971CCB 7275DDCB(普陀)(A)The Internet has led to a huge increase in credit-card fraud. Your card information could even be for sale in an illegal Web site. Web sites offering cheap goods and services

171、 should be regarded with care. On-line shoppers who enter their credit-card information may never receive the goods they bought. The thieves then go shopping with your card number or sell the information over the Internet.Computers hackers(黑客) have broken down security systems, raising questions abo

172、ut the safety of cardholder information. Several months ago,25 000 customers of CD Universe, an on-line music retailer(批发商),were not lucky. Their names, addresses and credit-card numbers were posted on a Web site after the retailer refused to pay US $157 828 to get back the information.Credit-card f

173、irms are now fighting against on-line fraud. Master Card is working on plans for Web-only credit card, with a lower credit limit. The card could be used only for shopping on-line.However, there are a few simple steps you can take to keep from being cheated.Ask about your credit-card firms on-line ru

174、les: Under British law, cardholders are responsible for the first US $78 of any fraudulent spending.And shop only at secure sites: Send your credit-card information only if the Web site offers advanced secure system.If the security is in place, a letter will appear in the bottom right-hand corner of

175、 your screen. The Web site address may also start with https:/the extra “s” stands for secure. If in doubt, give your credit-card information over the telephone.Keep your password safe: Most online sites require a user name and password before placing an order. Treat your passwords with care.65. Wha

176、t do most people worry about the Internet according to this passage?A. A lot of stolen credit-cards were sold on the Internet.B. Fraud on the Internet.C. Many Web sites are destroyed.D. Many illegal Web sites are on the Internet.66. What is the meaning of the word “fraud” ?A. cheating B. sale C. pay

177、ment D. safety67. How can the thieves get the information of the credit-card?A. The customers give them the information.B. The thieves steal the information from Web sites.C. The customers sell the information to them.D. Both A and B. 68. You are shopping on the site: ,and you want to buy a TV set,

178、what does this article suggest doing?A. Order the TV set at once.B. Do not buy the TV set on this site.C. E-mail the site your credit-card information.D. Tell the site your password and buy the TV set for you. (B)Before we send humans into deep space, well have to find out just how long the human bo

179、dy can survive in a weightless environment. One problem is that in space theres no physical sensation to let you know when youre upside down and astronauts have to rely on possible clues from their surroundings. A few hours after reaching orbit(轨道), one in three of all astronauts will experience spa

180、ce sicknessa feeling rather like carsickness. There is also the problem of protection from the extreme hazards(危险) of space such as constant meteorite bombardment(陨星轰击) and radiation. It is going to take some clever technology to keep rockets and astronauts from these dangers.Even with a well-protec

181、ted spaceship, space travel isnt going to be easy. No matter what you travel in, it is going to be a long flight in space! Science fiction writers often imagine using suspended animation(动画), a kind of forced long-term sleep, as a way for astronauts to escape the boredom of long missions.An even str

182、onger measure might be to freeze the astronauts. We already use cryogenic(低温) techniques to preserve dead bodies and store human embryos(胚胎). Freezing living adults may not be so far away, but perhaps we dont have to do that. Perhaps we should use our existing technology and send frozen embryos to t

183、he far corners of universe.Hundreds of years from now, billions and billions of miles away, the embryos will be thawed(解冻) and their hearts will start beating. These astronauts of the future will not grow inside a mothers body but will be produced in a machine. They will be brought up by robot. It m

184、ay seem strange but one day it might just happen.69. We can learn from the passage that _.A. scientists already use existing technology to store embryos for space flightB. cryogenic techniques will practically be used for space flight in hundreds of yearsC. it may be easier to freeze embryos than to

185、 freeze adults on the spaceshipD. most astronauts will suffer from space sickness70. In the future astronauts for extreme long journeys will grow up_.A. before they go into space B. during the space tripC. when they are inside mothers bodyD. after they return from space71. The passage is mainly abou

186、t _.A. the bright future of space flightB. the possibility to train embryo astronautsC. the physical and mental conditions of astronautsD. the problems and the possible solutions to them in long space flights ( C )NASDAQ, acronym for the National Association of Securities Dealers Automated Quotation

187、 system, is one of the largest markets in the world for the trading of stocks. The number of companies listed on NASDAQ is more than that on any of the other stock exchange in the United States, including the New York Stock Exchange(NYSE) and the American Stock Exchange (AMSE). The majority of compa

188、nies listed on NASDAQ are smaller than most of those on the NYSE and AMSX. NASDAQ has become known as the home of new technology companies, particularly computer and computer-related businesses. Trading on NASDAQ is started by stock brokers(经纪人) acting on behalf of their clients. The brokers negotia

189、te with market makers who concentrate on trading specific stocks to reach a price for the stock. Unlike other stock exchange, NASDAQ has no central location where trading takes place. Instead, its market makers are located all over the country and make trades by telephone and via the Internet. Becau

190、se brokers and market makers trade stocks directly instead of on the floor of a stock exchange, NASDAQ is called an over-the-counter market. The term over-the counter refers to the direct nature of the trading, as in a store where goods are handed over a counter. Since its inception(开市) in 1971, the

191、 NASDAQ Stock Market has been the innovator(创新者). As the worlds first electronic stock market, NASDAQ long ago set a precedent(先例) for technological trading innovation that is unrivaled(无对手的). Now ready to become the worlds first truly global market, the NASDAQ Stock Market is the market of choice f

192、or business industry leaders worldwide. By providing an efficient environment for raising capital NASDAQ has helped thousands of companies achieve their desired growth and successfully make the leap into public ownership.72. What is TRUE of NASDAQ?A. It is the largest stock market in the world.B. IT

193、 lists only small companies.C. It lists all the new technology companies.D. It lists the biggest number of companies.73. The word” negotiate”(Line 8,Para.1) means _. A. discuss B. argue C. interfere D. cope74. NASDAQ is also known as an over-the counter market because it seems_. A. a stock market B.

194、 a stock exchange C. a counter D. a store75. Since its founding in 1971, NASDAQ has contributed to _.A. raising money for many companies B. creating an efficient environment for many companiesC. the expected development of many companiesD. the unexpected development of many companies65-68 BABB 69-71

195、BBD 72-75DA DC(浦东)(A)You are walking along a lane and never know what you will find. It might be a shop selling oil paintings, or a place where you can buy Dutch cheese. One thing is for sure, you will end up by a canal.Welcome to Amsterdam, the capital of the Netherlands.The year 2013 will be a goo

196、d time to explore Amsterdam as the city has a lot to celebrate: the famous canal ring will turn 400 years old. Amsterdam will also celebrate the 160th birthday of Dutch painter of the Van Gogh museum, home to the biggest collection of the artists colorful works.The famous canal ring is a trademark o

197、f the city. With more than 100 kilometers of canals, it is no wonder that Amsterdam is called the Venice of the north. But unlike the Italian town, where boats were originally used for transport, the canals in Amsterdam were mainly used for defense in the 17th century. Today ferries on canals offer

198、a different way to explore the city.But to truly experience Amsterdam, you cant miss the citys historic museums and monuments, among which Van Gogh museum is the most famous one. It holds 200 paintings, 700 letters and 500 drawings by the artist. Another famous museum is Anne Franks house. Anne Fran

199、k became famous around the world because of a diary she left explaining her and her familys experience as a Jewish girl during World War II, hiding out in a house hoping that she would not be captured by German Nazis.Amsterdam is small enough to walk or cycle almost anywhere, but it is rarely dull.

200、Best of all it combines its glittering past with a rebellious edginess. 65. Which of the following is TRUE of the canals in Amsterdam? A. They had to be rebuilt after years of use. B. They protected the city in ancient times.C. They were built to provide transport for the city. D. They are the main

201、theme of Van Goghs paintings.66. In the passage, Amsterdam is described as a city _. A. that can be explored by footB. that is completely built on water C. that has two historic museums in all D. that is famous for its modernization67. What can we infer from the passage?A. People get lost easily in

202、Amsterdam. B. Van Gogh spent his life time in Amsterdam. C. Many Jews in Amsterdam were caught by Nazis. D. Taking a ferry ride is the most popular activity in Amsterdam.68. Where would this passage most probably appear? A. In a museum leaflet. B. In a history textbook. C. In an academic magazine. D

203、. In a travel brochure. (B)Which tablet computer should YOU be buying: They are this years must have. and theres a style to suit everyone?Best for young childrenLeapPad Explorer 2, 68Aimed at children between three and nine (though a nine-year-old might find it a little simple), it comes in pink or

204、blue and with five built-in education games (you can buy more). Besides, the LeapPad does not allow access to the internet so it is impossible for your child to stumble across anything inappropriate.Pros: The education games are well-designed, the built-in video camera is a fun way to play at being

205、a film director.Cons: Some of the games are shockingly expensive. And the power adaptor is not included.Best for teenagersiPad 4th generation, 399-659The iPad is still the market leader, and for good reason. If the teenager in your house enjoys playing computer games, the latest offering from Apple

206、is the one to choose.Pros: No other tablet can compete with the near one million apps (the name Apple created for specially-designed downloadable programs) available for the iPad.Simple to use, even for those who usually struggle with technology.Cons: Considerably more expensive than most competitor

207、s.Best forworking parentsMicrosoft Surface, 399-559Tablets are brilliant for leisure but what if you want to do a bit of work? No tablet can yet compete with a full-size laptop computer, but this is the only tablet that allows you to use Microsoft Word, Excel and Powerpoint (they are all pre-install

208、ed and included in the price) and you can buy a pretty lovely mini- keyboard for typing letters and emails, which also doubles up as the cover.Pros: The Surface is good for watching movies a bonus when stuck in the airport on a business trip and surfing the internet.Con: The keyboard is an expensive

209、 add-on costing up to 109. It might be cheaper to buy a laptop (though a tablet is much smaller and lighter).Best for bookwormsAmazon Kindle Paperwhite, 109Nearly all tablets let you download books. Its a great way to take a mountainous pile of hardbacks on holiday without stuffing your suitcase.But

210、 most tablets have a shiny screen which can be very distracting when youre trying to read. The Paperwhite is different: its matt screen and crisp black lettering imitate the look of words on paper brilliantly.And yet you can still read the words in the dark.Pros: Easy on the eye, excellent battery l

211、ife, 180,000 free books (if you subscribe to the Amazon Prime customer loyalty service) plus hundreds of thousands more to buy.Cons: No TV, films, games, internet or camera.69.The underlined phrase stumble across most probably means _.A. meet with B. quarrel with C. compare with D. compete with70. W

212、hich of the following about Surface is NOT TRUE?A. The keyboard will add to the cost.B. The keyboard can serve as a cover.C. You have to pay extra to install Microsoft Word.D. You can watch movies or surf the Internet with it.71. If you are a game lover, which tablet is least likely to be your choic

213、e?A. LeapPad Explorer 2. B. iPad 4th generation. C. Microsoft Surface. D. Amazon Kindle Paperwhite. (C)Seligman is leading the research on what might be called Happiness Revolution in psychology. Since World War II, psychologists have focused on fixing what is broken repairing psychosis(精神病), and ne

214、urosis(精神衰弱). Research has piled up steadily when it comes to looking at patients who are neurotic, while the happy or joyful people among us have received little scientific examination.When Seligman did a search to find academic articles about such positive psychology he found only 800 out of 70,00

215、0. Psychologists tend to be concerned with taking a negative 8 person, and helping him get to negative 2, said Seligman, a psychology professor of the University of Pennsylvania. My aim is to take a plus 2 person and boost him to a plus 6.In the last 50 years, statistics have shown that we are less

216、happy as a people. While our quality of life has increased dramatically over that time, and weve become richer, were in an epidemic of depression, Seligman said. Depression is 10 times more common now, and life satisfaction rates are down as well. Seligman argues that the new science he writes about

217、 is shifting psychologys model away from its narrow-minded focus on mental illness towards positive emotion, virtue and strength that increase peoples happiness. If you want to be happy, forget about winning the lottery(抽奖), getting a nose job, or securing a raise.In his new book, Authentic Happines

218、s, psychologist Martin Seligman argues that overall lifetime happiness is not the result of good genes, money, or even luck. Instead, he says we can increase our own happiness by making use of the strengths and virtues that we already have, including kindness, originality, humor, optimism, and gener

219、osity. He has named the field Positive Psychology, arguing that we would be better off building on our own strengths rather than mourning, and, hence, trying to repair, our weaknesses. By frequently calling upon their strengths, people can build up natural barriers against misfortune and negative em

220、otions, he said.Science has shown that there are several distinct roads to being a happy person though happiness might not mean what you think. Material goods even simple ones like ice cream, and massages are only stimulating things that rapidly give people a boost.To cultivate happiness, we must fi

221、rst find out our individual strengths and virtues. Next, apply the qualities in such a way as to enhance your happiness-generating system.72. The distinctive feature of Seligmans work lies in _.A. evaluating the psychological state of peopleB. making a study of people who suffer from mental illnessC

222、. focusing the scientific examination on the happy or joyful people D. figuring out the exact number of the academic articles about positive psychology 73. What does Seligman mean by saying take a plus 2 person and boost him to a plus 6?A. We should focus on happy or joyful people.B. We need more an

223、d more happy and joyful people.C. Its difficult to make people happy from a plus 2 to plus 6.D. Happy people also need to improve their level of happiness.74. According to the passage, which of the statements might Martin Seligman support?A. Promotion leads to true happiness.B. We can rely on our st

224、rengths for happiness.C. Intelligent people are usually more satisfied.D. Mental illness should be the focus of psychology.75. The passage mainly talks about _.A. strengths and happinessB. ways to cultivate happinessC. development of psychologyD. effect of positive psychology6568. BACD6971. ACD7275.

225、 CDBB(闵行)(A)(You may read the questions first)Below is a web page fromGrammy AwardsBeyonc and Taylor Swift are ready to take home a lot of hardware tonight as they lead all artists with 10 and eight nominations (提名). The ladies also perform, along with fellow Album of the Year nominees the Black Eye

226、d Peas, Lady Gaga and the Dave Matthews Band. In addition, Bon Jovi, Green Day, Lady Antebellum, Maxwell, Pink and Zac Brown Band will also take the stage. But perhaps the most expected moment is the Michael Jackson tribute (颂词), which features the 3-D mini-movie “Earth Song” and will be highlighted

227、 by performers Celine Dion, Jennifer Hudson, Smokey Robinson, Carrie Underwood and Usher.MaurerA mysterious company named Sabre, headed by an attractive, stubborn CEO (Kathy Bates), appears out of nowhere and buys out Dunder Mifflin. As Michael is unwilling to follow Sabres new policies, Andy and Er

228、in write a welcome song to greet the company. Meanwhile, Pam and Jim hope to get accepted into a desired local day-care center.Surviving SurvivorIts hard to believe its been 10 years since the first season of Survivor. But tonight ten of the games most iconic players, including previous winners Rich

229、ard Hatch, Parvati Shallow and Tom Westman, discuss the series in detail and provide understanding into how the game is actually played and the impact its had on their lives. In addition, updates on beloved former castaways, like Ethan Zohn, Rupert Boneham and Elisabeth Hasselbeck, are given.Private

230、 PracticeSo Mayas pregnant and wants to have the baby. What to do now? Dink (Stephen Lunsford), the boy who did his part to get her into this mess, has a solution: He wants to marry her. You can imagine how Sam and Naomi will feel about that. And if that wont raise thehistrioniclevel enough for you,

231、 how about this: A couple has two dying daughters, and only one can be saved. Their choice? And Brian Benben is back, so expect more Pete-Violet- Sheldon worry.65.Suppose Edna is crazy about pop stars, which programme is she most likely to watch?A. Grammy AwardsB. MaurerC. Surviving SurvivorD. Priva

232、te Practice66.According to the passage, Surviving Survivor is most probably_.A. a musical playB. a game showC. a love storyD. a cultural documentary67. The word “histrionic” in the passage probably means “_”.A. amusingB. seriousC. uniqueD. dramatic(B)Ridgewood is a small, quiet town 20 miles from Ma

233、nhattan. It is a typical suburban town, perfect for raising children away from the fast pace of the city. However, some Ridgewood kids feel as upset as if they were on the citys busy streets. In addition to hours of homework, Ridgewoods children are occupied with afterschool activities- from swimmin

234、g to piano to religious classes.Out of desperation one day, the town decided to schedule another activity. This one was called “Ridgewood Family Night - Ready, Set, Relax!” Instead of schedules filled with sports, music, or overtime at the office, some of the towns 25,000 residents decided to take t

235、he night off and stay home. For a few months before Family Night, a committee of volunteers worked hard to spread the word. Younger students took “Save the Date for Me” leaflets home to their parents. The mayor issued a statement, and schools and clubs agreed to cancel homework and meetings so famil

236、ies could relax and be together.The tension between a hope for a more relaxed lifestyle and the knowledge that the benchmark for success has been raised in recent years weighs heavily on the minds of the townspeople. Some parents like to recall a different kind of childhood, one without so many sche

237、duled afterschool activities. However, these same parents feel obliged to make sure their children are prepared to survive in todays high-pressure work environment. They are afraid that any gap in their childrens physical or intellectual development might mean they wont be admitted to the “right” un

238、iversities and wont succeed in a more and more competitive world.Nevertheless, it seems that Family Night worked, at least to a point. Cars moved easily around Ridgewoods normally busy downtown streets, and stores and restaurants saw a drop in business. Some families ate supper together for the firs

239、t time in months.Initially, there was great hope of taking back their lives. But sadly, few families believe that one night will change their lives. Many are sure that they will fall back into the habit of over-scheduling their children to be overachieving adults.68.The writer describes Ridgewood as

240、 a town where _.A. residents take little notice of educationB. children are stressful and over expectedC. parents are concerned about childrens safetyD. children lead a life of relaxed, ordinary rhythms69.According to the passage, which of the following isNOTtrue about the Family Night?A. Schools as

241、signed no homework.B. Fewer people ate in the restaurants.C. The streets were less crowded.D. People enjoyed meeting each other.70.According to the passage, the parents in Ridgewood _.A. have conflicting desires about what experiences to offer their childrenB. are happy with the lifestyle they have

242、chosen for themselves and the childrenC. believe that it is their duty to make a change for the stressful life styleD. are worried about their childrens lacking of big ambition71.We may get the conclusion from the passage that _.A. the concept of Family Night will become a regular part of life in Ri

243、dgewoodB. Ridgewood people believe the Family Night will change their way of livingC. Family Night is not as popular as people have originally thoughtD. schools are worried about students being given too much free time(C)Although websites such as Facebook and MySpace experienced increasing growth du

244、ring the middle of the first decade of the 21st century, some users fail to realize the fact that the information they post online can come back to trouble them. First, employers can monitor employees who maintain a blog, photo diary, or website. Employers can look for troublesome employees opinions

245、, sensitive information disclosures, or wildly inappropriate conduct. For example, a North Carolina newspaper fired one of its features writers after she created a blog on which she anonymously wrote about the characteristics of her job and coworkers.The second unintended use of information from soc

246、ial networking websites is employers who check on potential employees. A New York Times article reported that many companies recruiting (招募) on college campuses use search engines and social networking websites such as MySpace, Xanga, and Facebook to conduct background checks. Although the use of My

247、Space or Google to investigate a students background is somewhat upsetting to many undergraduates, the Times noted that the use of Facebook is especially shocking to students who believe that Facebook is limited to current students and recent alumni.Company employers are not the only people interest

248、ed in college students lives. The third unintended use of social networking websites is college administrators who monitor the Internetespecially Facebookfor student misconduct. For example, a college in Bostons Back Bay expelled (除名) its Student Government Association President for joining a Facebo

249、ok group highly critical of a campus police sergeant. In addition, fifteen students at a state university in North Carolina faced charges in court for underage drinking because of photos that appeared on Facebook.Although more users of websites such as Facebook are becoming aware of the potential pi

250、tfalls (陷阱) of online identities, many regular users still fail to take three basic security precautions. First, only make your information available to a specific list of individuals whom you approve. Second, regularly search for potentially harmful information about yourself that may have been pos

251、ted by mistake or by a disgruntled (不满的) former colleague. Third, never post offensive material under your name or on your page as, despite the best precautions, this material will likely make its way to the wider world. By taking these simple steps, members of the digital world can realize the many

252、 benefits of e-community without experiencing some of the damaging unintended consequences.72.The main purpose of the passage is to _.A. explain the growth of the digital world through from the aspect of privacyB. discuss the risks of the digital world and suggest ways to protect yourselfC. propose

253、steps Facebook, MySpace, and Google can take to protect user privacyD. illustrate potential unintended uses of private information73.The author implies that users should take all of the following actions to protect their online privacy EXCEPT_.A. know to whom you make your online information availab

254、leB. avoid uploading information that would be damaging if it were discoveredC. carefully select and limit who can view your electronic profileD. speak with employers to inform them of any misinformation published about you74.Which of the following best describes the relationship between the last pa

255、ragraph and the other paragraphs?A. It offers detailed examples to support previous points of view.B. It summarizes the points of view in the previous paragraphs.C. It provides suggestions to solve the previously mentioned problems.D. It offers evidence to make the previous points of view clearer.75

256、.According to the passage, which of following does the writer imply?A. Information obtained unwillingly from the Internet is permissible in court.B. It is impossible to protect yourself from unintended uses of information online.C. Even if you restrict who can view your data, the government may stil

257、l access it.D. Done properly, posting offensive information about oneself causes no risk.6568 ABDB 6971 DAC 7275 BDCA(静安)(A)14 DAYS FROM JUST 2,090! FULLY INCLUSIVE FROM THE UKPrice cover international airfares, departure taxes, fuel charge, local transportation all meals, entrance fees, guides, dai

258、ly tours and visas for the UK citizens. Days 1-3: UK ShanghaiFly to great city of Shanghai and in the evening sample traditional Shanghai food. Visit the beautiful YU Garden, Old Town. Shanghai Museum, cross the Great Nanpu Bridge and tour the Pudong area. Also explore Xintiandi with its 1920s style

259、 Shikumen buildings and end your stay inShanghai with an amazing Huangpu River evening tour. Days 4-7: Shanghai Yangtze River TourFly to Yichang and change (approx: on hour) to board your Yangtze River ship for the next four nights. Enjoy a tour of the three Gorges Dam (三峡大坝)before sailing on the gr

260、and Yangtze River, passing through the impressive Three Gorges. We take a side trip to the lesser Three Gorges or travel up the Shennong Stream in a peapod boat and enjoy various shore trips along the way. Day 8: Chongqing ChengduGet off in Chongqing and drive to Chengdu for an overnight stay. Days

261、9-10: Chengdu XianVisit the famous Panda Reserve to see the lovely animals. We then fly to the historic city of Xian for two nights stay and enjoy traditional Shuijiao. Next day, explore one of the most important discoveries of the 20th century the Terracotta Warriors (兵马俑),followed by the ancient C

262、ity Wall and a performance of Tang Dynasty dancing. Days 11-13: Xian BeijingVisit little wild Goose Pagoda and see the ancient objects at the well-known Shaanxi provincial museum before walking through the lively Muslim Quarter to see the Great Mosque. Later fly to Beijing for three nights stay and

263、try Peking Duck. During our stay in Beijing, we stroll through Tiananmen Square to the Forbidden City, the largest and best preserved collection of ancient buildings in China, and visit the Summer Palace. Next day we take a walk on the Great Wall, tour the unique Temple of Heaven and enjoy an attrac

264、tive Chinese Acrobatic Show. Day 14: Beijing UKFly back to the UK, arriving home later the same day filled with happy memories.65. The underlined word sample in the passage probably means _.A. buyB. reserveC. tasteD. make66. The first and last scenic spots they visited in Xian are _.A. the Terracott

265、a Warriors and the Great MosqueB. the Terracotta Warriors and Shanghai Provincial MuseumC. little wild Goose Pagoda and Great MosqueD. little wild Goose Pagoda and the Muslim Quarter67. Which of the following is TRUE according to the ad. ?A. The tourists will have to pay extra for fuel and meals.B.

266、The tourists will visit the 1920s Shikumen buildings in Beijing.C. The tourists will take a side trip to the Three Gorges Dam during the tour. D. The tourists will stay in Beijing for three nights before leaving for the UK.68. The ad. is mainly intended to _.A. encourage the British to travel in Chi

267、na B. attract the British to traditional Chinese foodC. offer service of booking air tickets to touristsD. provide the British with a better understanding of Shanghai(B)There are two factors which determine an individuals intelligence. The first is the sort of brain he is born with. Human brains dif

268、fer considerably, some being more capable than others. But no matter how good a brain he has to begin with, an individual will have a low order of intelligence unless he has opportunities to learn. So the second factor is what happens to the individual 一 the sort of environment in which he is reared

269、. If an individual is handicapped environmentally, it is likely that his brain will fail to develop and he will never attain the level of intelligence of which he is capable.The importance of environment in determining an individuals intelligence can be demonstrated by the case history of the identi

270、cal twins, Peter and Mark. Being identical, the twins had identical brains at birth, and their growth processes were the same. When the twins were three months old, their parents died, and they were placed in separate foster homes. Peter was reared by parents of low intelligence in an isolated commu

271、nity with poor educational opportunities. Mark was reared in the home of well-to-do parents who had been to college. He was read to as a child, sent to good schools, and given every opportunity to be stimulated intellectually. This environmental difference continued until the twins were in their tee

272、ns, when they were given tests to measure their intelligence. Marks I.Q. was 125, twenty-five points higher than the average and fully forty points higher than his identical brother. Given equal opportunities, the twins, having identical brains, would have tested at roughly the same level.69. Accord

273、ing to the passage, the average I.Q. is _.A. 85 B. 100 C. 110 D. 125 70. This passage suggests that an individuals I.Q. _. A. can be predicted at birth B. is inherited from his parentsC. stays the same throughout his life D. can be increased by education 71. This selection can best be titled_. A. Me

274、asuring Your Intelligence B. Intelligence and Environment C. The Case of Peter and Mark D. How the brain Influences Intelligence (C)At some time in your life you may have a strong desire to do something strange or terrible. However, chances are that you dont act on your impulse, but let it pass inst

275、ead. You know that to commit the action is wrong in some way and that other people will not accept your behavior.Perhaps the most interesting thing about the phenomenon of taboo (禁忌的) behavior is how it can change over the years within the same society, how certain behavior and attitudes once consid

276、ered taboo can become perfectly acceptable and natural at another point in time. Topics such as death, for example, were once considered so upsetting and unpleasant that it was a taboo to even talk about them. Now with the publication of important books such as On Death and Dying and Learning to Say

277、 Goodbye, people have become more aware of the importance of expressing feelings about death and, as a result, are more willing to talk about this taboo subject.One of the newest taboos in American society is the topic of fat. Unlike many other taboos, fat is a topic that Americans talk about consta

278、ntly. Its not taboo to talk about fat; its taboo to be fat. The “in” look is thin, not fat. In the work world, most companies prefer youthful-looking, trim executives to sell their image as well as their products to the public. The thin look is associated with youth, vigor, and success. The fat pers

279、on, on the other hand, is thought of as lazy and lacking in energy, self-discipline, and self-respect. In an image-conscious society like the U.S., thin is “in”, fat is “out”.Its not surprising, then, that millions of Americans have become obsessed (着迷) with staying slim and “in shape”. The pursuit

280、of a youthful physical appearance is not, however, the sole reason for Americas obsession with diet and exercise. Recent research has shown the critical importance of diet and exercise for personal health. As in most technologically developed nations, the life-style of North Americans has changed dr

281、amatically during the course of the last century. Modern machines do all the physical labor that people were once forced to do by hand. Cars and buses transport us quickly from point to point. As a result of inactivity and disuse, peoples bodies can easily become weak and vulnerable to disease. In a

282、n effort to avoid such a fate, millions of Americans are spending more of their time exercising every day.72. From the passage we can infer taboo is.A. a strong desire to do something strange or terribleB. a crime committed on impulse committedC. behavior considered unacceptable in societys eyesD. a

283、n unfavorable impression left on other people73. Based on the ideas presented in the passage we can conclude being fatA. will always remain a tabooB. is not considered a taboo by most peopleC. has long been a tabooD. may no longer be a taboo some day74. In the U.S., thin is “in”, fat is “out”, this

284、means.A. thin is “inside”, fat is “outside”B. thin is “devoted”, fat is “lazy”C. thin is “youthful”, fat is “spiritless”D. thin is “fashionable”, fat is “unfashionable”75. The main reason the passage gives for why so many Americans are exercising regularly is _.A. their changed life-styleB. their ea

285、gerness to stay thin and youthfulC. their appreciation of the importance of exerciseD. the encouragement they have received from their companies75. The main reason the passage gives for why so many Americans are exercising regularly is _.A. their changed life-styleB. their eagerness to stay thin and

286、 youthfulC. their appreciation of the importance of exerciseD. the encouragement they have received from their companiesSection B 6568 CADA 6971 BDB 7275 CDDB(金山)(A)A major new development in system of work in Britain is taking place. Flexible working hours, or Flextime, are catching on fast, and tr

287、end is continuing. In 1973, over 500 organizations had adopted the idea, and by 1974, this number had risen to over 200,000.Flexible working hours were invented in Germany in the late 1960s, but reached Britain only in 1972. The system allows workers to start and finish work whenever they want, with

288、 only two requirements. These are, firstly, that all workers must present for certain key times in the day, and secondly, that all workers must work an agreed total number of hours per week.The system had proved a total success wherever it has been tried. A survey of 700 workers on flexible hours sh

289、owed three main advantages: a better balance between working and private life, avoidance of the need to travel during rush hours and the ability to finish a certain task before leaving. From the employers point of view, the system tends to increase productivity, reduce labor hour turnover and give w

290、orkers a greater sense of duty. At first, Flexible was mainly confined to white collar workers, but it is now applied to manual workers too.65. According to Flextime system, workers need not_ .A. work at the same timeB. work all the weekdaysC. work hard D. ask for leave when being absen66. “Key” tim

291、e is a period when _.A.visitors come to the plantsB. all workers must be at workC.employers go round in the workshopsD. rush hours are over67. No matter where it is used, this system has proved_.A. entirely effectiveB. totally correctC. a complete failureD. quite difficult68. One of the main advanta

292、ges of “Flextime” for workers is that they _.A. have a greater sense of dutyB. can avoid busy trafficC. can get higher payD. can avoid working hard(B)Fastest Time to Run 100 Miles on a Treadmill (跑步机)The fastest time to run 100 miles on a treadmill by a team (of 12) is claimed at 9hr 5 min 17 sec by

293、 a team consisting of staff and prisoners at the Young Offenders Institute in Wigan to raise money for Children in Need.Largest Game of Chinese WhispersSadly, the record attempt for the largest game of Chinese Whispers was unsuccessful on 13 November 2008. The existing record of 1330 children still

294、remains. Fastest Mile with an Egg/Spoon in both handsThe fastest mile egg and spoon race with both hands is 8 min 25 sec and was achieved by Ashrita Furman (USA) in an attempt broadcast by ESPN 260, at Disneys Wide World of Sports, in Orlando, Florida, USA, on 13 November 2008. Longest Mexican WaveT

295、he record for the Longest Mexican Wave was attempted by 250,000 people on the streets of Adelaide on 8 November 2008 sadly, the wave was not completed by all participants and the attempt was unsuccessful. The current record of 157, 574 remains.Largest Kebab (烤肉串)The longest kebab measures 2047.47 m

296、(1.27 miles) and was achieved by the ArcelorMittal Newcastle Works on occasion of the companys annual Community Day, in Newcastle, South Africa.69. The above information is _. A. a series of sports events B. a list of broken and unbroken records C. a report of funny sports items D. a description of

297、challenging human limits 70. Which of the following can NOT be found in the information? A. The countries where the attempts were made. B. The persons who made the attempts. C. The reasons why some attempts failed. D. The present record for each item. 71. Which item was attempted individually? A. Fa

298、stest Time to Run 100 Miles on a Treadmill B. Largest Game of Chinese Whispers C. Fastest Mile with an Egg/Spoon in both hands D. Longest Mexican Wave( C )Do you know how it is when you see someone yawn and you start yawning too? Or how hard it is to be among people laughing and not laugh yourself?

299、Well, apparently its because we have mirror neurons (神经元)in our brains. Put simply, the existence of mirror neurons suggests that every time we see someone else do something, our brains imitate (模仿)it, whether or not we actually perform the same action. This explains a great deal about how we learn

300、to smile, talk, walk, dance or play sports. But the idea goes further: mirror neurons not only appear to explain physical actions, they also tell us that there is a biological basis for the way we understand other people. Mirror neurons can undoubtedly be found all over our brains, but especially in

301、 the areas which relate to our ability to use languages and to understand how other people feel. Researchers have found that mirror neurons relate strongly to language. A group of researchers discovered that if they gave people sentences to listen to (for example: The hand took hold of the ball), th

302、e same mirror neurons were triggered as when the action was actually performed (in this example, actually taking hold of a ball). Any problems with mirror neurons may well result in problems with behavior. Much research suggests that people with social and behavioral problems have mirror neurons whi

303、ch are not fully functioning. However, it is not yet known exactly how these discoveries might help find treatments for social disorders. Research into mirror neurons seems to provide us with even more information concerning how humans behave and interact (互动). Indeed, it may turn out to be the equi

304、valent (相等物)for neuroscience of what Einsteins theory of relativity was for physics. And the next time you feel the urge to cough in the cinema when someone else doeswell, perhaps youll understand why. 72. Mirror neurons can explain _. A. why we cry when we are hurt B. why we cough when we suffer fr

305、om a cold C. why we smile when we see someone else smile D. why we yawn when we see someone else stay up late 73. The underlined word triggered in the third paragraph probably means “_”. A. set off B. cut off C. built up D. broken up 74. We can learn from the passage that mirror neurons _. A. relate

306、 to human behavior and interaction B. control human physical actions and feelings C. result in bad behavior and social disorders D. determine our knowledge and language abilities 75. What is the passage mainly about? A. Ways to find mirror neurons. B. Problems of mirror neurons. C. Existence of mirr

307、or neurons. D. Functions of mirror neurons.65 68 ABAB 69 71 BCC 72 75 CAAD (嘉定) AWork your brain!Between 2 and 3 pounds of wonder, it controls everything we say, do or think, who we are and what we care about, the way we walk or laugh or figure out things, what we like and the talents we possess, ho

308、w we see and talk and run and jump and process our food.The brain uses 20 percent of our bodys oxygen and 20 percent of its blood. Somewhere within its protein, fat, 100,000 miles of blood vessels and 100 billion nerve cells, it helps us remember where we put our gym shoes. Change our temperature so

309、 we dont die because of the heat or cold. Speed us up or slow us down. Help us choose between orange juice or orangeflavored drinks.Its complexity are stunning, far beyond anything most of us can imagine. To keep this work of art as polished as possible we need to eat right, exercise and keep mental

310、ly stimulated.Good nutrition helps brain cells communicate with each other. Exercise stimulates a hormone in our brain that improves memory. Mental stimulation keeps you sharp even as you age.“Its very important that we tell people to be physically active and mentally active,” said neurologist Malco

311、lm Stewart.“People cannot stop aging, but youre able to reduce the damage; youre able to keep the function up.”Following are Dr.Stewarts advice for improving brain health:l NutritionAvoid fast food. Follow the old adage(格言): For breakfast, eat like a king; for lunch, like a queen; for supper, like a

312、 beggar.l ExerciseDo a combination of stretching aerobic and musclestrengthening every day.l Mental gamesTry to have a sense of hope about the future. Do puzzles. Listen to music. Reach out to others to make their lives better.65. The purpose of the author in writing this passage is to _.A. inform u

313、s how the brain works B. give us advice on how to keep the brain healthy C. tell us that the brain plays an important role in our lives D. show how special the brain is to us 66. The word “stunning” in Paragraph 5 means _.A. interesting B. strange C. significant D. amazing 67. According to the text,

314、more exercise _.A. keeps our mind sharp B. helps improve our memory C. gives our brain a rest D. is good for brain cells communicating with each other 68. In order to keep brain healthy,we should avoid _.A. eating a good lunch B. doing puzzles C. eating a large supper D. taking aerobic exercise BBlo

315、gs are here to stay. They have become a deep-rooted part of the literary, journalistic, and even social area.Perhaps you would like to start a blog; perhaps you would like to improve your blog; perhaps you would like to take your blog to professional standardsas a means of earning incomes or increas

316、ing your visibility. Whatever your goal is, this course will move you in the right direction.Millions of people have blogs. Only a part of them do it well enough to achieve real benefits, in terms of incomes, visibility, or sheer (纯粹的) entertainment. This course will show you how to join the band of

317、 better bloggers. You will learn how to find the right tone for your blog, set it up and maintain it, write posts that are effective and worth reading, and attract people from all over the world to your spot on the blogsphere. While its simple to start a blog, it takes vigilance and skill to do it r

318、eally well, as is true of any kind of writing.Among the topics covered: overviews of the blogsphere, the benefits of blogging, focusing on your blog concept, how to set up and maintain a blog, tips for effective blog writing and drawing visitors to your site.You will get hands-on practice creating y

319、our blog plus time for questions. Whether you wish to blog for fun or business, you will gain a realistic understanding of how to become part of the blogsphere.Course offered.HOW TO BLOGIn this four-week course, a blogging expert will guide you through the process of conceiving, setting up, writing,

320、 and promoting a blog. Each student will consult with the instructor on a blog concept, then set up a “live” blog and introduce it to the rest of the class for feedback. Students will also have ample opportunities to ask the instructor questions.ONLINE CLASSESTuition: $125. Registration Fee: $25Each

321、 class is strictly limited to 25 students, aged 18 and up.StartsNew LectureLocationTimeEnrollJan 12WednesdaysOnline24hrs. a dayCLASS FULLFeb 9WednesdaysOnline24hrs. a dayCLASS FULLMar16WednesdaysOnline24 hrs. a dayEnroll69. The passage is mainly about_.A. an online programme about blog history B. an

322、 online writing arrangementC. an online course for writing good blogs D. an online competition in writing blogs70. After joining in the course, you will_.A. learn skills of writing effective blogsB. have to recite the blog conceptsC. be required to write blogs as a careerD. choose blogs for fun or b

323、usiness71. What do we know about the course?A. There are no more than 25 students in each course.B. The course is intended for teenagers under 18.C. It will cost you $150 to learn the course.D. New lectures are given each day every week.CAs we have seen, the focus of medical care in our society has

324、been shifting from curing disease to preventing diseaseespecially in terms of changing our many unhealthy behavior, such as poor eating habits, smoking, and failure to exercise. The line of thought involved in this shift can be pursued further. Imagine a person who is about the right weight, but doe

325、s not eat very nutritious foods, who feels OK but exercises only occasionally, who goes to work every day, but is not an outstanding worker, who drinks a few beers at home most nights but does not drive while drunk, and who has no chest pains or abnormal blood counts(血细胞计数), but sleeps a lot and oft

326、en feels tired. This person is not ill. He may not even be at risk for any particular disease. But we can imagine that this person could be a lot healthier.The field of medicine has not traditionally distinguished between someone who is merely “not ill” and someone who is in excellent health and pay

327、s attention to the bodys special needs. Both types have simply been called “well.” In recent years, however, some health specialists have begun to apply the terms “well” and “wellness” only to those who are actively striving to maintain and improve their health. People who are well are concerned wit

328、h nutrition and exercise, and they make a point of monitoring their bodys condition. Most important, perhaps, people who are well take active responsibility for all matters related to their health. Even people who have a physical disease or handicap (缺陷) may be “well,” in this new sense, if they mak

329、e an effort to maintain the best possible health they can in the face of their physical limitations. “Wellness” may perhaps best be viewed not as a state that people can achieve, but as an ideal that people can strive for. People who are well are likely to be better able to resist disease and to fig

330、ht disease when it strikes. And by focusing attention on healthy ways of living, the concept of wellness can have a beneficial impact on the ways in which people face the challenges of daily life. 72. Today medical care is placing more stress on _.A. keeping people in a healthy physical conditionB.

331、monitoring patients body functionsC. removing peoples bad living habitsD. ensuring peoples psychological well-being73. In the first paragraph, people are reminded that _.A. good health is more than not being illB. drinking, even if not to excess, could be harmfulC. regular health checks are essentia

332、l to keeping fitD. prevention is more difficult than cure74. Traditionally, a person is considered “well” if he _.A. does not have any unhealthy living habits B. does not have any physical handicapsC. is able to handle his daily routinesD. is free from any kind of disease75. According to the author,

333、 the true meaning of “wellness” is for people _.A. to best satisfy their bodys special needsB. to strive to maintain the best possible healthC. to meet the strictest standards of bodily healthD. to keep a proper balance between work and leisure65-68 BDBC 69-71 C A C 72-75 CADB(黄浦)(A) Most people who

334、 live in Seattle, Washington, love their city. There is a never-ending flow of fun things to do. But, people who live there do not always enjoy the day-after-day absence of sunshine during the winter months. It is a documented fact that sunshine (or lack of it) plays a major role in how each of us m

335、eets the day. It also affects how we perform at school or work. When people are deprived of sunlight, they can develop seasonal affective disorder, which makes it difficult for them to feel happy or get things accomplished. No major city in the United States is more affected by the “sunshine factor”

336、 than Seattle.To fight drizzle and fog during the winter months, Steve Murphy created a business that is very popular among the locals. The Indoor Sun Shoppe is more than a little ray of sunshine during the gray days of Seattles winter. His shop offers a huge source of plants and artificial lighting

337、 for people who are trying to overcome seasonal affective disorder. Located in Fremont, Washington, The Indoor Sun Shoppe has an amazing collection of exotic (奇异的) plants and “good bugs” in a humid and well-lit environment. At The Indoor Sun Shoppe, you can spend up to $400 for artificial lighting t

338、hat will chase away the winter blues. It will also keep your plants healthy. You can buy a dawn simulator(模拟器) that will gradually fill your room with a warming wake-up glow. What better way to meet a Seattle morning when real sunshine is nothing more than a happy thought!Murphys in-home waterfalls

339、are also popular with customers. But his plants and lights remain the “main course.” On a cloudy winter day, The Indoor Sun Shoppe is a bright spot in Seattle!65. Which of the following statements is NOT TRUE according to the passage?A. Seattle is located in the state of Washington.B. Citizens love

340、Seattle for their colorful city lives. C. Sunshine in Seattle is rare during the winter months. D. People who live in Seattle are used to the sunshine absence.66. Which of the following best defines the term seasonal affective disorder in Para.2?A. mild temperature and a general feeling of illness c

341、aused by gray winterB. exhaustion, depression and lack of energy caused by a lack of sunlightC. winter-related illness caused by the low temperature and little sunlightD. post-holiday depression that comes when relatives leave after Christmas67. The Indoor Sun Shoppe _.A. attracts people from differ

342、ent parts of the whole countryB. aims to create a humid and well-lit environment during winter monthsC. brings only a little ray of sunshine during the gray days of Seattles winterD. offers variety of goods helping people overcome seasonal affective disorder 68. Which of the following best explains

343、the use of a dawn simulator?A. An alarm clock stimulated by the light rays of early morning.B. A bringer of indoor artificial sunshine when there is none outside.C. An automatic waterfall system helping make indoor plants healthy.D. A better way to meet a Seattle morning with real warming sunshine.

344、(B)Hotel InformationThe following activities are offered for guests of the Hong Kong Disneyland Hotel. If you have questions about these programs, please call Guest Services by dialing “0”. Highlights Around the Hong Kong Disneyland ResortHotel ActivitiesPrince Eric Activities RoomOpening Hours:9:00

345、am-10:00pm(Close 15 minutes for cleaning every 2 hours)Maximum Capacity:22Disney-themed kids activities are available to offer our little guests unlimited fun and magical experience.Victorian Spa Exclusive Offer30% off Elemis Modern Skin Facial(Original Price:$1,280)Designed to do with skin care con

346、cernsRebalances sebum level/Reduces redness/For bookings, please call 3510 6388 Activity time:10:30 am-9:30pmLittleKidsBigKidsMickeys PaintingDress up our Disney characters with your imagination and give them a colourful life! LittleKidsBigKidsTeensSun VisorBlocking the sun is wise to enjoy your hol

347、iday! Come and make your own sun visor(帽舌)!TeensBigKidsIQ Fun ZoneTurn on your mind engine and show your teamwork to complete the missions!LittleKidsActivity time:9:45 pm-10:00pmBedtime StoryJoin our Recreation Hosts who recount tales from the magical world of Disney!TeensBigKids Age3-7 Age8-12 Age1

348、3-17 Mickey MazeWith views of the South China Sea, our hedge maze in the shape of Mickey Mouse is a great place to while away an afternoon. Youll find the Mickey Maze directly outside of the Grand Salon. Jogging TrailsMeasured jogging trails of varying distances have been identified throughout the H

349、ong Kong Disneyland Resort. You can pick up a jogging map at the Victorian Spa or at the Front Desk. Inspiration Lake Recreation CentreInspiration lake is 34 hectares and is also one of the largest public parks in Hong Kong. Its a great place to rest and relax! You can choose from indoor fitness sta

350、tions and relax under the shade of gazebos, enjoy bike rentals and pedal boat rentals outside. LittleKidsIn-house hotel guests could enjoy 50% discount for renting one hour pedal boat at ILRC.All activities and additional information are subject to change without notice.69. Guests of Hong Kong Disne

351、y Hotel can _.A.enjoy Elemis Modern Skin Facial at the price of $ 384B. call Guest Services by dialing “0” to book Victoria Spas exclusive offerC. spend a whole day viewing the South China Sea in Mickey MazeD. choose from different jogging routes according to a reliable jogging map 70. Inspiration L

352、ake Recreation Centre _. A. attracts guests as one of the largest public parks in DisneylandB. is a right place for in-house guests to reduce redness C. provides guests with both indoor and outdoor activitiesD. offers all guests 50% off for renting one hour pedal boat71. According to the information

353、 about Prince Eric Activities Room, we can learn that_.A. there is about an hour and 15 minutes for cleaning everyday B. at least 22 quests can take part in different activities there at a time C. both kids and teens can feel free to create their own sun visors thereD. IQ Fun Zone helps children bel

354、ow 7 to cultivate their team spirit(C) Throughout the centuries, various writers have contributed greatly to the literary treasure trove of books lining the shelves of todays libraries. In addition to writing interesting material, many famous writers, such as Edgar Allan Poe, were larger-than-life c

355、haracters with personal histories that are as interesting to read as the stories they wrote. Poes rocky life included being driven off from the United States Military Academy at West Point in 1831 and an ongoing battle with alcohol. Yet, despite heavy gambling debts, poor health, and terrible unempl

356、oyment, Poe managed to produce a body of popular works, including “The Raven” and “The Murders in the Rue Morgue.”Herman Melville, author of Moby Dick, once lived among the man-eaters in the Marquesas Islands and wrote unusual tales inspired by his years of service in the U.S. Navy. Dublin-born Osca

357、r Wilde was noted for his charismatic personality, his outrageous lifestyle, and creating witty catchphrases such as, “Nothing succeeds like excess.” D.H. Lawrence wrote shameful novels that were often cut, and Anne Rice led a double life writing bestselling horrible novels under her real name and u

358、sing “A.N. Roquelaure” for the lowbrow unhealthy novels she penned on the side. Nonconformist(不墨守成规的)author and naturalist Henry David Thoreau once fled to the woods and generated enough interesting material to fill his noted book Walden. Thoreau wrote on the issue of passive resistance protest in h

359、is essay “Civil Disobedience” and served time in jail for refusing tax payments in protest of the United States governments policy towards slavery. American short story writer O. Henrys colorful life was ruined by tragic events, such as being accused and sentenced for stealing money from an Austin,

360、Texas bank. Despite his success selling his short stories, O. Henry struggled financially and was nearly bankrupt when he died. As diverse as these famous authors backgrounds were, they all led unconventional lives while writing great literary works that will endure throughout the ages. The next tim

361、e you read an interesting book, consider learning more about the author by reading his or her biography so you can learn about the unique life experiences that shaped his or her writing. 72. Which of the following is NOT TRUE about Edgar Allan Poe? A. He had his own interesting personal history as o

362、ther famous writers. B. He graduated from United States Military Academy at West Point in 1831. C. He once had heavy gambling debts and fell down with illness. D. He managed to produce popular works no matter how rocky his life was. 73. What can you infer from Para.2?A. D.H. Lawrence and Anne Rice o

363、nce wrote similar type of novels. B. Anne Rice used a pen name because her novels were quite popular.C. Anne Rice used different names when she wrote bestselling novels. D. Henry David Thoreau enriched his books by living in the forests. 74. Which statement is TRUE according to the passage?A. Henry

364、David Thoreau was passionately opposed to horrible events. B. Anne Rice always used a pen name to conceal her true identity. C. Herman Melville stayed with man-eaters during his naval service. D. O Henrys life was colorful though he had lots of difficulties. 75. The passage mainly tells us that _. A

365、. many famous writers lived nontraditional lives B. most famous writers were usually troublemakers C. writers had to lead interesting lives to generate materialsD. the biographies of famous writers are always inspirational6568DBDB6971DCC7275BACA(虹口)(A)A new study of 8,000 young people in the Journal

366、 of Health and Social Behavior shows that although love can make adults live healthily and happily, it is a bad thing for young people. Puppy love (早恋) may bring stress for young people and can lead to depression (忧郁症). The study shows that girls become more depressed than boys, and younger girls ar

367、e the worst of all.The possible reason for the connection between love and higher risk of depression for girls is “loss of self”. According to the study, even though boys would say “lose themselves in a romantic relationship”, this “loss of self” is much more likely to lead to depression when it hap

368、pens to girls. Young girls who have romantic relationships usually like hiding their feelings and opinions. They wont tell that to their parents.Dr Marian Kaufman, an expert on young people problems, says 15% to 20% young people will have depression during their growing. Trying romance often causes

369、the depression. She advises kids not to jump into romance too early. During growing up, it is important for young people to build strong friendships and a strong sense of self. She also suggests the parents should encourage their kids to keep close to their friends, attend more interesting school ac

370、tivities and spend enough time with family.Parents should watch for signs of depression - eating or mood changes - and if they see signs from their daughters or sons, they need to give help. The good news is that the connection between romance and depression seems to become weak with age. Love will

371、always make us feel young, but only maturity (成熟) gives us a chance to avoid its bad side effects.65. Whats the main idea of the passage?A. Puppy love may bring young people depression.B. Parents should forbid their childrens love.C. Romance is a two edged sword for adults. D. Romance is good for yo

372、ung people.66. Which of the following is more likely to have depression?A. Young people who have a strong sense of selfishness.B. Young boys whose parents watch for their behavior.C. Young girls who always hide their feelings and opinions.D. Careless parents whose children are deep in love.67. What

373、can be inferred from the passage?A. Lacking love can lead young people to grow up more quickly.B. Early love makes young people keep close to their friends and parents.C. Parents should help their children to be aware of the signs of depression.D. The older a woman is,the less likely she seems to lo

374、se herself in romance.68. Whats the authors attitude towards puppy love?A. Confused. B. Disapproving. C. Disinterested. D. Scared. (B)It helps to choose the colors you plan to live with in your home with awareness of how different colors affect your emotional and physical states. Every color has a u

375、nique meaning, and this can significantly affect how you feel in a room.Simple Solution: Using this simple chart, you can choose a rooms color to be in balance and harmony with the role you want a room to have in your and your familys life.Red: A room that is painted red will energize, inspire activ

376、ity and movement, help passion, and be powerful and stimulating. A red room can also be exhausting and overpowering, and stressful for those who are anxious.Orange: Orange rooms encourage happy, joyful, social gatherings. While an orange dining room or family room can stimulate the appetite, it is a

377、 great choice for such gathering places. Orange enhances parties, communication, positive feelings, and general good cheer.Yellow: Yellow rooms inspire intellectual clarity, organization, clear-headed thinking, and happy energy. Yellow is considered a color for the logical left side of the brain. Ye

378、llow is also very bright and somewhat energizing.Blue: Blue rooms are often chosen for bedrooms and meditation (冥想) rooms because its cool energy is very calming, restful, peaceful, and spiritual. Blue helps inspire a quiet meditative quality, and color therapy with blue has been found to reduce blo

379、od pressure. Blue is also useful to make one to sleep.Green: Green rooms are very restful but combine with an energizing quality. Green is the color of outdoors, calm and active at the same time. Green brings balance and harmony to a room, and can be used as a calming place for people who are troubl

380、ed or in need of refreshment.69. What impression can green rooms give to you?A. Passionate and powerful.B. Refreshing and harmonious.C. Cool and calming.D. Inspiring and bright.70. If you are going to hold a family gathering, which color of the room can best enhance your atmosphere?A. Red.B. Orange.

381、C. Blue.D. Green.71. Which of the following statements about color is NOT true?A. Yellow rooms inspire logical and clear thinking.B. Blue creates a quietmeditative quality.C. Different colors affect you emotionally and physically.D. Red rooms are acceptable when you are in a bad mood. (C)Prolonging

382、human life has increased the size of the human population. Many people alive today would have died of childhood diseases if they had been born 100 years ago. Because more people live longer, there are more people around at any given time. In fact, it is a decrease in death rates, not an increase in

383、birthrates, that has led to the population explosion.Prolonging human life has also increased the dependency load. In all societies, people who are disabled or too young or too old to work are dependent on the rest of society to provide for them. In hunting and gathering cultures, old people who cou

384、ld not keep up might be left behind to die. In times of famine (饥荒), infants might be allowed to die because they could not survive if their parents starved, where as if the parents survived they could have another child. In most contemporary societies, people feel a moral obligation (责任) to keep pe

385、ople alive whether they can work or not. We have a great many people today who live past the age at which they want to work or are able to work; we also have rules which require people to retire at a certain age. Unless these people were able to save money for their retirement, somebody else must su

386、pport them. In the United States many retired people live on social security checks which are so little that they must live in near poverty. Older people have more illnesses than young or middle-aged people; unless they have wealth or private or government insurance, they must often “go on welfare”

387、if they have a serious illness.When older people become senile (衰老的) or too weak and ill to care for themselves, they create grave problems for their families. In the past and in some traditional cultures, they would be cared for at home until they died. Today, with most members of a household worki

388、ng or in school, there is often no one at home who can care for a sick or weak person. To meet this need, a great many nursing homes and convalescent (疗养的) hospitals have been built. These are often profit making organizations, although some are sponsored by religious and other nonprofit groups. Whi

389、le a few of these institutions are good, most of them are simply “dumping (倒垃圾的) grounds” for the dying in which “care” is given by poorly paid, overworked, and under skilled personnel (员工).72. The author believes that the increased size of the human population results from _.A. an increase in birth

390、rates B. the industrial developmentC. a decrease in death rates D. childhood diseases 73. It can be inferred from the passage that in hunting and gathering cultures _.A. it was a moral responsibility for the families to keep alive the aged people who could not workB. the survival of infants was less

391、 important than that of their parents in times of starvationC. old people were given the task of imparting (传授) the cultural wisdom of the tribe to new generationsD. death was celebrated as a time of rejoicing for an individual freed of (免除) the hardships of life 74. The phrase “this need” in the la

392、st paragraph refers to _. A. prolonging the dying old peoples livesB. reducing the problems caused by the retired peopleC. making profits through caring for the sick or weak peopleD. taking care of the sick or weak people75. According to the passage, which of the following statements about the old p

393、eople in the United States is true?A. Many of them live on social security money which is hardly enough.B. Minority of them remain in a state of near poverty after their retirement.C. Many nursing homes are sponsored by religious and nonprofit groups.D. Older people have wealth or private insurance

394、if they have a serious illness.65-68: ACDB69-71: BBD72-75: CBDA(奉贤)(A)She turned up at the doorstep of my house in Cornwall. No way could I have sent her away. No way, not me anyway. Maybe someone had kicked her out of their car the night before. Were moving house. “No space for her any more with th

395、e baby coming. We never really wanted her, but what could we have done? She was a present. People find all sorts of excuses for abandoning an animal. And she was one of the most beautiful dogs I had ever seen.I called her Goldie. If I had known what was going to happen I would have given her a more

396、creative name. She was so unsettled during those first few days. She hardly ate anything and had such an air of sadness about her. There was nothing I could do to make her happy, it seemed. Heaven knows what had happened to her at her previous owners. But eventually at the end of the first week she

397、calmed down. Always by my side, whether we were out on one of our long walks or sitting by the fire.Thats why it was such a shock when she pulled away from me one day when we were out for a walk. We were a long way from home, when she started barking and getting very restless. Eventually I couldnt h

398、old her any longer and she raced off down the road towards a farmhouse in the distance as fast as she could. By the time I reached the farm I was very tired and upset with Goldie. But when I saw her licking the four puppies I started to feel sympathy towards them. We didnt know what had happened to

399、her, said the woman at the door. I took her for a walk one day, soon after the puppies were born, and she just disappeared. She must have tried to come back to them and got lost, added a boy from behind her.I must admit I do miss Goldie, but Ive got nugget now, and she looks just like her mother. An

400、d Ive learnt a good lesson: not to judge people.65. How did the author feel about Goldie when Goldie came to the house?A. Shocked.B. Sympathetic. C. Annoyed.D. Upset.66. In her first few days at the authors house, Goldie_.A. felt worried B. was angryC. ate a littleD. sat by the fire67. Goldie rushed

401、 off to a farmhouse one day because she _.A. saw her puppiesB. heard familiar barkingsC. wanted to leave the authorD. found her way to her old house68. The passage is organized in order of _.A. timeB. effectiveness C. importanceD. complexity(B)69. Probably most injuries happen to skaters wrists beca

402、use _.A. few skaters wear wrist guardsB. the wrist is the most fragile (脆弱的) body partC. skaters often push out their hands when fallingD. skaters dont know how to protect their wrists70. _ are the majority in the study.A. Those who wore helmetsB. Those who wore wrist guardsC. Those who wore knee pa

403、dsD. Those who wore no protective gear71. What was the main point of the study?A. To encourage people to learn skating.B. To advise skaters to wear safety pads.C. To advertise for skating protective gear.D. To introduce ways of protecting skaters.(C)Almost a decade after the hit Finding Nemo made cl

404、ownfish seem totally warm and lovable, environmentalists are now looking for a real-life sequel: Saving Nemo. In the United States, a request has been made to extend the protections of the Endangered Species Act to marine species including the clownfish. But before you start shedding tears for Nemo

405、and his buddies, keep in mind that this request is not based on any evidence of a decline in the clownfish population. Instead, what has sparked concern is the worsening health of coral reefs, which more than one million aquatic species including the clownfish depend on to thrive, even survive. In w

406、ays it makes more sense to move to protect a species when its habitat declines rather than its actual population. The most important mission of the Endangered Species Act is the protection of species habitats; without their habitats, theres almost no hope of saving endangered animals, except perhaps

407、 in a zoo. Earlier this year, alarming news came about the worlds largest coral reef system, Australias Great Barrier Reef (GBR, 大堡礁). A study has shown the Reef is in sharp decline, with half of its coral cover gone in the past 27 years. Katharina Fabricius, an Australian coral reef ecologist co-au

408、thoring the study, has been diving and working on the GBR since 1988and has watched the decline. “There are still a lot of fish . but not the same color and diversity as in the past,” she said. Thestudyteamusedinformationfrommorethan 2,000 surveys to determine the rate of decline between 1985 and 20

409、12. That overall 50-percent decline, they estimate, is a yearly loss of about 3.4 percent of the Reef. If the trend continued, the coral cover could halve again by 2022. Several main factors are responsible for the decline, the study found. Intense tropical cyclones (热带气旋), believed to be fueled by

410、global warming, have caused massive damage to reefs in the central and southern parts of the Reef. Meanwhile, population explosions of the coral-consuming crown-of-thorns starfish (长棘海星) have affected coral populations along the length of the Reef. Two severe coral bleaching (变白) events, caused by o

411、cean warming, have also had major damaging impacts in northern and central parts of the GBR.72. A request has been made in the United States to save the clownfish because_.A. it is a totally warm and lovable species B. the health of its habitat is worsening C. its population has dropped sharplyD. fe

412、w clownfish can be found in zoos72. What does the underlined word “thrive” in Paragraph 3 probably mean? A. To feed oneself.B. To keep oneself safe.C. To grow and develop well.D. To attract attention.74. According to the article, what is the AUTHORS attitude toward the request in the U.S.?A. He/she

413、supports it.B. He/she finds it ridiculous.C. He/she thinks it is reasonable but needs revising.D. Theres no WAY to tell.75. According to the article, all of the following factors contribute to the decline of Great Barrier Reefs coral cover except _.A. the growing popularity of scuba (水肺)diving aroun

414、d the ReefB. stronger tropical cyclones fueled by global warmingC. the rapidly growing population of type of coral-eating starfishD. coral bleaching caused by rising temperature of the ocean waterA篇 6568 BADAB篇6971 CDBC篇7275 BCAA(宝山)(A)Lighter and cheaper than high-quality video cameras, todays best

415、 smartphones can shoot and edit high-definition (HD) videos. With lower-quality lenses(透镜) and image sensors, smartphones probably wont replace professional video cameras anytime soon. Still, some creative filmmakers are extending the borders of smartphone moviemaking by shooting professional-qualit

416、y films using only smartphones.Soon after Apples iPhone 4 was released, two filmmakers decided to make a short film using only their iPhone 4s. Michael Koerbel and Anna Elizabeth James shot and edited the one-and-a-half-minute film Apple of My Eye in only 48 hours. Audiences were fascinated with the

417、 film, in which a man and his grandfather connect emotionally while admiring a model train set in a store window. The filmmakers followed up their success with Goldilocks, an extraordinary spy novel told over nine, three-minute films.Many filmmakers are convinced that smartphone films are here to st

418、ay. One sign is their inclusion in major film festivals. In February 2012, organizers of the Berkshire International Film Festival staged the 10 X 10 On North Festival. Entries included Oliver by Hooman Khalili, an award-winning film about a girl whose special powers enable her to brighten the lives

419、 of three lonely people. Another entry was Yearlapse11, a 365-second film by Zsolt Haraszti that describes an actual journey he made from New York to London.During the festival, which ran from February 16 to 26, these and many other smartphone films were shown at the Beacon Cinema in Pittsfield, Mas

420、sachusetts. Similar festivals in Canada and South Korea have given smartphone filmmakers opportunities to show off their creative work.65. What does the article imply about smartphone filmmakers? A. Their efforts have gotten little attention. B. Their biggest challenge is finding actors. C. Theyve i

421、nfluenced famous movie studios D. They must overcome equipment limitations.66. What does the article point out about Harasztis film? A. It uses slow-motion effects. B. It took just minutes to make. C. Its cast was quite large. D. Its based on true events.67. Which film is about secret agents? A. App

422、le of My Eye. B. Goldilocks. C. Olive. D. Yearlapse 11.68. According to this article, what is true about the festivals in Canada and South Korea? A. Theyre held at the same time of year. B. Theyre part of an emerging trend. C. They both receive government support. D. Theyre still in the planning sta

423、ges.(B) We Are Now Open Daily thru Columbus Day The museum and store are open from 10 a.m. to 5 p.m. throughout the season. Trolley rides start with the first departure at 10:05 a.m. and continue until the end of the day with the last ride being at 4:15 p.m. We are open Rain or Shine!April 30 to May

424、 21May 28 to October 10October 15 to October 30Open Saturdays and Sundays OnlyOpen Every DayOpen Saturdays and Sundays OnlyIce Cream & Sunset Trolley Ride Special ( $3.50 per person )At 7:30 p.m., every Wednesday and Thursday evening in July and August, join us for a special sunset trolley ride with

425、 ice-cream included. Bring the whole family!Regular FaresSenior Citizens:$5.50Adults:$7.50Children (Ages 616):$5.00Children 5 and under:Free Discounts are available for tour groups.Admission includes unlimited trolley rides, access to the Museum Grounds and Exhibits, and use of our picnic areas to e

426、njoy your own picnic lunch.69. From now on to Columbus Day, the museum is open _. A. only on Sundays and Saturdays B. on sunny days C. every day D. all of the above70. If a family of 4 persons one senior, a couple and a 12-year-old boy are taking a sunset trolley ride, how much is the total admissio

427、n fee? A. $18. B. $25.5. C. $33. D. $39.5.71. Which one of the following is NOT included in the admission? A. Unlimited sunset trolley ride. B. Access to the Museum Grounds. C. A visit to the Museum Exhibits. D. A use of the museum picnic areas. ( C )Discoveries in science and technology are thought

428、 by “untaught minds” to come in blinding flashes or as the result of dramatic accidents. Sir Alexander Fleming did not, as legend would have it, look at the mold ( 霉 ) on a piece of cheese and get the idea for penicillin there and then. He experimented with antibacterial substances for nine years be

429、fore he made his discovery. Inventions and innovations almost always come out of tough trial and error. Innovation is like soccer; even the best players miss the goal and have their shots blocked much more frequently than they score. The point is that the players who score most are the ones who take

430、 most shots at the goaland so it goes with innovation in any field of activity. The prime difference between innovation and others is one of approach. Everybody gets ideas, but innovators work consciously on theirs, and they follow them through until they prove practicable or otherwise. What ordinar

431、y people see as fanciful abstractions, professional innovators see as solid possibilities. “Creative thinking may mean simply the realization that theres no particular goodness in doing things the way they have always been done.” Wrote Rudolph Flesch, a language authority. This accounts for our reac

432、tion to seemingly simple innovations like plastic garbage bags and suitcases on wheels that make life more convenient: “How come nobody thought of that before?” The creative approach begins with the proposal that nothing be as it appears. Innovators will not accept that there is only one way to do a

433、nything. Faced with getting from A to B, the average person will automatically set out on the best-known and apparently simplest route. The innovator will search for alternate courses, which may prove easier in the long run and are sure to be more interesting and challenging even if they lead to dea

434、d ends. Highly creative individuals really do march to a different drummer.72. What does the author probably mean by “untaught mind” in the first paragraph? A. An individual who often comes up with new ideas by accident. B. A person who has had no education. C. A citizen of a society that restricts

435、personal creativity. D. A person ignorant of the hard work involved in experimentation.73. According to the author, what differs innovators from non-innovators? A. The way they present their findings. B. The way they deal with problems. C. The intelligence they possess. D. The variety of ideas they

436、have.74.The phrase “march to a different drummer” (the last line of the passage) suggests that highly creative individuals are _. A. unwilling to follow common ways of doing things B. diligent in pursuing their goals C. concerned about the advance of society D. devoted to the progress of science75.The most suitable title for this passage might be _. A. The Relation Between Creation and Diligence B. To Be a Creative Expert in the Study of Human Creativity C. What Are So Special about Creative IndividualsD. Discoveries and Innovation6568 DDBB 6971 CBA 7275 DBAC

展开阅读全文
相关资源
猜你喜欢
相关搜索

当前位置:首页 > 幼儿园

Copyright@ 2020-2024 m.ketangku.com网站版权所有

黑ICP备2024021605号-1